UPSC OCTOBER - 2019 DAILY CURRENT AFFAIRS CLASS TEST - 01 2. Consider the following pairs: 1. With reference to the Constitution of India, Festivals State consider the following statements: 1. Losar - Arunachal Pradesh 1. Article 130 says that, the Supreme Court 2. Bathukamma - Chhattisgarh shall sit in Delhi or in such other place or 3. Wari - Maharashtra places, decided by President of India, on Which of the pairs given above is/are correct advice of Prime Minister. matched? 2. In Kerala, Supreme Court has its regional a) 1 only bench established long back in b) 1 and 3 only since 1968. c) 2 and 3 only Which of the statements given above is /are d) 1, 2 and 3 correct? Answer - B a) 1 only Explanation b) 2 only Statement 2 is wrong c) Both 1 and 2 Bathukamma is celebrated in Telangana. d) Neither 1 nor 2 Why this Question: Answer- D Telangana government recently celebrated the Explanation: eagerly awaited ‘Festival of Flowers’ Statement 1 is Wrong (Bathukamma) for which the State is known. Article 130 says that “the Supreme Court shall sit Bathukamma. in Delhi or in such other place or places, as the The festival was traditionally celebrated by Chief Justice of India may, with the approval of women across the state. the President, from time to time, appoint.” Every year this festival is celebrated as per Telugu It is decided by Chief Justice of India and approval version of Hindu calendar in the Bhadrapada of President is needed. Amavasya, also known as Mahalaya Amavasya, Statement 2 is also Wrong usually in September–October of Gregorian There is no regional bench of supreme court in calendar. Kerala. In Telugu, ‘Bathukamma’ means ‘Mother Goddess Why this Question: come Alive’. Recently VP M Venkaiah Naidu made the It is the state festival of Telangana. following suggestions;

Institute four regional Benches to tackle the enormous backlog of cases, and to ensure their speedy disposal.

The court should be split into two divisions.

www.iasgatewayy.com

UPSC OCTOBER - 2019 DAILY CURRENT AFFAIRS CLASS TEST - 01

3. Consider the following statements with Choose the Correct Option: respect to Sendai Framework: a) 1 only 1. It is a voluntary, non-binding agreement b) 2 only on disaster risk reduction. c) 1 and 5 only 2. It is the successor of Hyogo Framework d) 3 and 5 only for Action. Answer: A 3. India is a signatory to Sendai National Ambient Air Quality Standards are the Framework. standards for ambient air quality set by the Which of the given statement/s is/are correct? Central Pollution Control Board (CPCB) that is a) 1 and 2 only applicable nationwide. b) 2 and 3 only The CPCB has been conferred this power by the c) 1, 2 and 3 Air (Prevention and Control of Pollution) Act, d) 2 and 3 only 1981. Answer: C The measurement of air quality is based on eight Sendai Framework for Disaster Risk Reduction pollutants, namely, PM5, sulphur dioxide (SO2), 2015-2030 or “Sendai Framework” is an nitrogen dioxide (NO2), PM10, lead (Pb), carbon international accord on disaster risk reduction, monoxide (CO), Ozone, Ammonia (NH3). adopted at the Third UN World Conference in 5. Consider the following about Sendai, Japan, on March 18, 2015 as the successor Pattamaadai mat. instrument to the Hyogo Framework for Action 1. Pattamaadai mats are silk mats that 2005-2015. It is a 15-year, voluntary, non-binding originated in a small village in agreement. Andra Pradesh India is a signatory. 2. The have been given GI tag The United Nations Office for Disaster Risk Which of the above statement(s) is/are Incorrect? Reduction is tasked with the implementation and a) 1 only follow up of the Sendai Framework. b) 2 only Topic: National Ambient Air Quality Standards c) Both 1 and 2

d) Neither 1 nor 2 4. Which of these pollutants are NOT included Answer: A in measurement of air quality under the Explanation: The Geographical Indications National Ambient Air Quality Standards? Registry has granted the Geographical Indication 1. Carbon dioxide (GI) tag to Pattamaadai Silk Mat 2. Carbon monoxide Pattamaadai mats are silk mats also called Pattu 3. Ammonia paai that originated in a small village in 4. Lead Tirunelveli district of Tamil Nadu. 5. Ozone www.iasgatewayy.com

UPSC OCTOBER - 2019 DAILY CURRENT AFFAIRS CLASS TEST - 01 6. Consider the following statements about 3. We run into a deficit if the value of the Saraswati Samman annual award: goods and services we import exceeds the 1. It is given for outstanding prose or poetry value of those we export. literary works in any of the 22 languages Which of the above statement(s) is/are correct? of India listed in Schedule VIII of the a) 1 only Constitution of India. b) 2 only 2. It was initiated by Ministry of Culture, c) 2 and 3 only d) All the above India Answer: D Which of the above statement(s) is/are correct?

a) 1 only 8. Consider the following statements about b) 2 only Index of Industrial Production (IIP) c) Both 1 and 2 1. It is a composite indicator measuring d) Neither 1 nor 2 changes in the volume of production of a Answer: A basket Explanation: The Saraswati Samman is an 2. of industrial products over a period of annual award for outstanding prose or poetry time, with respect to a chosen base literary works in any of the 22 languages of India period. listed in Schedule VIII of the Constitution of 3. Currently, the base year is 2010-11. India. 4. It is computed and published by the The Saraswati Samman was instituted in 1991 by Central Statistical Office the K. K. Birla Foundation. It consists of Rs 15 Which of the above statement(s) is/are correct? lakh, a citation and a plaque. a) 1 only Candidates are selected from literary works b) 2 only published in the previous ten years by a panel that c) 1 and 3 only included scholars and former award winners. d) All the above

Answer: C 7. Consider the following statements about Explanation: Index of Industrial Production Current Account Deficit (IIP) is a composite indicator measuring changes 1. The current account measures the flow of in the volume of production of a basket of goods, services and investments into and industrial products over a period of time, with out of the country. respect to a chosen base period. 2. The current account includes net income, Currently, the base year is 2011-12. including interest and dividends, and It is computed and published by the Central transfers, like foreign aid. Statistical Office (an office under the Ministry of Statistics and Programme Implementation) on

www.iasgatewayy.com

UPSC OCTOBER - 2019 DAILY CURRENT AFFAIRS CLASS TEST - 01 monthly basis with a time lag of six weeks from of 30 critical indicators that assess the delivery of the reference month. quality education.

Statement 3 is wrong: Kerala (1), Rajasthan 9. Consider the following statements about (2), Karnataka(3), Andhra Pradesh (4), Gujarat School Education Quality Index (SEQI) (5), Assam (6), Maharashtra (7), Tamil Nadu (8), 1. It is developed to evaluate the Himachal Pradesh (9), Uttarakhand (10), Haryana (11), Orissa (12), Chhattisgarh (13), performance of States and Union Telangana (14), Madhya Pradesh (15), Jharkhand Territories (UTs) in the higher education (16), Bihar (17), Punjab (18), Jammu Kashmir (19) sector. and Uttar Pradesh (20). 2. It is developed by NITI Aayog in

collaboration with Ministry of Human Resource Development. 10. Consider the following statements about 3. According to the report Kerala has paleochannel:

topped in terms of its performance 1. Paleochannel, is a remnant of an inactive river or stream channel that has been filled followed immediately by Tamil Nadu and or buried by younger sediment. Andhra Pradesh 2. It typically acts as pathways for Which of the statement(s) given above is/are groundwater movement and provide a correct? potential source of groundwater.

a) 1 only Which of the above statement(s) is/are correct? b) 2 and 3 only a) 1 only c) 2 only b) 2 only d) 1, 2 and 3 c) Both 1 and 2 Answer: C d) Neither 1 nor 2

Explanation: Answer: C

Statement 1 is wrong:

It is developed to evaluate the performance of States and Union Territories (UTs) in the school education sector and not higher education sector

Statement 2 is correct: The index will serve as a regular and transparent review of the status of school education quality across the States and UTs. Developed through a collaborative process including key stakeholders such as MHRD, the World Bank and sector experts, the index consists

www.iasgatewayy.com

UPSC OCTOBER - 2019 DAILY CURRENT AFFAIRS CLASS TEST - 02 3. Identify the correct statement about the 1. Identify the correct statement about the School Education Quality Index (SEQI): Sovereign Gold Bond Scheme: 1. It was released by the NITI Aayog to a) It is launched under the Gold Monetisation assess learning outcomes. Scheme 2. It does not assess the state of b) RBI Notifies the terms and conditions for infrastructure of schools. the scheme Choose the correct answer: c) Both are correct a) 1 only d) Both are incorrect b) 2 only Answer: C c) Both 1 and 2 Explanation: Sovereign Gold Bond Scheme was d) Neither 1 nor 2 launched under Gold Monetisation Scheme. Answer: A The issues are made open for subscription in Explanation: SEQI was developed by NITI tranches by RBI in consultation with GOI. Aayog to evaluate the performance of States and RBI Notifies the terms and conditions for the UTs in the school education sector. scheme from time to time. It is developed through a collaborative process, The rate of SGB will be declare by RBI before including key stakeholders such as Ministry of every new tranche by issuing a Press Release. HRD, the World Bank and sector experts.

It also assesses the Infrastructure and facilities for 2. National e Governance Division (NeGD) outcomes. falls under,

a) Ministry of Electronics and Information 4. Which of the following is/are advantages of Technology using plastic waste in National Highways b) Ministry of Communications construction? c) Department of Administrative Reforms 1. Higher resistance to deformation and Public Grievances d) Ministry of Statistics and Programme 2. Improved stability and strength Implementation 3. Reduced consumption of bitumen Answer: A Choose the correct answer: Explanation: National eGovernance Division a) 1, 2 and 3 (NeGD) falls under the Ministry of Electronics b) 2 only and Information Technology (MeitY). c) 1 and 3 only d) 1 and 2 only Answer: A Explanation: Plastic waste has already been used in wearing courses of National Highways

www.iasgatewayy.com

UPSC OCTOBER - 2019 DAILY CURRENT AFFAIRS CLASS TEST - 02 construction on the pilot basis in the states of 7. Global Hunger Index is released by Tamil Nadu and Kerala. a) World Health Organisation Construction of 1 kilometre of 4-lane highway can b) Economic Intelligence Unit help in disposal of approximately 7 tonnes of c) Food and Agriculture Organisation waste plastic. d) Welthungerhilfe foundation The advantages are higher resistance to Answer: D deformation, Increased durability and improved fatigue life and Improved stability and strength. 8. Consider the following pairs with respect to This also reduces consumption of bitumen in the festivals and their states. mixes. Besides, this process opens up an avenue 1. Moatsu - Nagaland for environment friendly disposal of waste plastic. 2. Chapchar kut - Mizoram

5. Which one of the following index is NOT 3. Mopin - Arunachal Pradesh published by NITI AAYOG? Which of the statements given above is correct? a) India Innovation Index a) 1 only b) SDG India Index b) 1 and 2 only c) ASER Index c) 1 and 3 only d) School Education Quality Index (SEQI) d) All the above Answer: C Answer: D Explanation: The Annual Status of Education, ASER report by the NGO Pratham which 9. Consider the following with respect to measures the learning outcomes has been International Astronomical Union (IAU). released. The report was based on 1. It was founded in 1919 and it later an annual household survey which covered 5.46 formed a part of United Nations Group. lakh children in 596 rural 2. It acts as the internationally recognized districts across the country. authority for assigning designations and All other Index are publish by NITI AAYOG. names to celestial bodies.

3. Its members includes only Nation states. 6. Economic and Technical Cooperation Which of the statements given above is Agreement which is recently seen in the correct? news is proposed between India and which a) 1 and 2 only of the following, b) 2 only a) United States c) 2 and 3 only b) European Union d) All the above c) Sri Lanka Answer: B d) South

Answer: C

www.iasgatewayy.com

UPSC OCTOBER - 2019 DAILY CURRENT AFFAIRS CLASS TEST - 02 10. Consider the following with respect to E- cigarettes: 1. They are non-combustible, battery- operated device that produces aerosol by heating the solution that it contains. 2. They are completely good for the health as they are nicotin free products. Which of the statements given above is Incorrect? a) 1 only b) 2 only c) Both 1 and 2 d) Neither 1 nor 2 Answer: B

www.iasgatewayy.com

UPSC OCTOBER - 2019 DAILY CURRENT AFFAIRS CLASS TEST - 03 1. Recently the term Coupled Forecast Model Delhi, Himachal Pradesh, Madhya Pradesh, (CFS) has been in news. It refers to Chhattisgarh, Haryana and Rajasthan a) New monsoon prediction model The directive was issued under Section 5 of The deployed by IMD Environment (Protection) Act, 1986. b) Economic model to predict an economic recession 3. The “Paryatan Parv” is an initiative of c) IMF’s new lending model a) Ministry of Tourism d) None of the above b) Ministry of External Affairs c) Ministry of Home Affairs Answer: A d) None of the above Explanation: The new monsoon model, called Answer: A the Coupled Forecast Model (CFS), deployed by the IMD under the National Monsoon Mission th (NMM) has failed to forecast the excess rainfall 4. On the 150 birth Anniversary of Mahatma received during Aug-Sept 2019. Gandhi, an innovation challenge which provides a platform for every child across 2. The Ganga Basin states include India to ideate innovative solutions for a 1. Uttarakhand sustainable India of their dreams, using 2. UP Gandhi’s principles. - The Gandhian 3. Bihar Challenge was launched by 4. Jharkhand 1. NITI Aayog’s Atal Tinkering Labs Choose the correct answers from above 2. UNICEF India a) 1 ,2 and 3 only 3. Generation Unlimited b) 2, 3 and 4 only Choose the correct answers from above c) 1,3 and 4 only a) 1 and 3 only d) All the above b) 2 and 3 only Answer: D c) 1 and 2 only Explanation: The Central government has d) All the above issued a 15-point directive, including cordoning Answer: D off ghats and imposing a fine of Rs 50,000, to Explanation: On the 150th birth Anniversary of prevent the immersion of idols in the Ganga or its Mahatma Gandhi, AIM, NITI Aayog’s Atal tributaries during festivals, including Dussehra, Tinkering Labs (ATL) and UNICEF India, Diwali, Chhath and Saraswati Puja. The directive including Generation Unlimited, have launched has been issued by the National Mission for Clean ‘The Gandhian Challenge’. Ganga (NMCG) to chief secretaries in 11 Ganga basin states. The 11 Ganga basin states include:

Uttarakhand, UP, Bihar, Jharkhand, West Bengal,

www.iasgatewayy.com

UPSC OCTOBER - 2019 DAILY CURRENT AFFAIRS CLASS TEST - 03 5. With reference to Drone Regulation in India, their services towards the cause of the consider the following statements elderly persons. 1. The regulation defines the “No Drone 3. It is awarded in two categories – Zones”. Government Institutions and 2. The Drone regulation is drafted by Individuals from any part of the country. Ministry of Home Affairs. Which of the statements given above is/are 3. DigiSky is a portal for registration and incorrect? flying of civil drones in India. a) 3 only Which of the statements given above is/are b) 2 and 3 only correct? c) 1 and 3 only a) 1 only d) 1 and 2 only b) 2 and 3 only Answer: A c) 1 and 3 only Explanation: It is awarded in two categories – d) 1, 2 and 3 Institutions and Individuals from any part of the Answer: C country. Explanation: Statement 2 is wrong. Regulations of Drone is drafted by Ministry of 8. Swachh Rail, Swachh Bharat Report, 2019 Civil Aviation and Not by Ministry of Home was released by Affairs. Statement 1 & 3 are correct. a) Niti Aayog b) Ministry of Railways 6. In Koppen scheme of classification, India’s c) Ministry of Urban Development Tropical Monsoon is denoted by d) None of the above a) Af Answer: B b) Aw c) Am 9. Recently, Drone Innovators Network d) Cs Summit-2019 was held in New Delhi. It was Answer: C organized by

7. Consider the following statements about a) World Economic Forum Vayoshreshtha Samman b) World Bank 1. It is a Scheme of awards instituted by the c) IMF Ministry of Social Justice & d) Asian Development Bank Answer: A Empowerment and it has status of

National Awards.

2. It is conferred on the eminent senior

citizens and institutions in recognition of

www.iasgatewayy.com

UPSC OCTOBER - 2019 DAILY CURRENT AFFAIRS CLASS TEST - 03

10. Maitree – 2019, is a military exercise between a) India and Myanmmar b) India and Thailand. c) India and Bangladesh d) India and Nepal Answer: B Explanation: Maitree – 2019.It is the joint military exercise between India and Thailand. It was recently conducted in Meghalaya.

www.iasgatewayy.com

UPSC OCTOBER - 2019 DAILY CURRENT AFFAIRS CLASS TEST - 04 1. Consider the following statemtns about 3. Which of the following states have launched “Audio Odigos” recently in news: the “Mo Sarkar” programme? 1. It’s a personal audio guide app to explore a) Gujarat the stories behind many iconic Indian b) Maharashtra heritage sites. c) Odisha 2. While the app will include English, d) Rajasthan Hindi, Japanese, Korean, it is not Answer: C available in vernacular languages. Explanation: Odisha Chief Minister Naveen Which of the above statement(s) is/are correct? Patnaik launched his government’s new a) 1 only governance initiative ‘Mo Sarkar’ on the occasion b) 2 only of Gandhi Jayanti. The objective of the ‘Mo c) Both 1 and 2 Sarkar’ programme is to provide service with d) Neither 1 nor 2 dignity to people who are coming to government Answer: A offices for different purposes. Explanation: The app will include English, Hindi, Japanese, Korean and other vernacular 4. Consider the following pairs: languages. 1. Dampa Tiger Reserve: Mizoram 2. Gumti Wildlife Sanctuary: Sikkim 2. Recently an initiative called “Parichay” is in 3. Saramati Peak: Nagaland news, what is it? Which of the above pairs is/are correctly a) An initiative for easy filing of RTI through matched? an online portal a) 1 only b) New Secondary School examination portal b) 2 and 3 only c) A collaborative clearing house of litigation c) 1 and 3 only and research assistance for lawyers filing d) 1, 2 and 3 appeals against exclusion from the NRC Answer: C d) None of the Above Explanation: Dampa Tiger Reserve, the largest Answer: C wildlife sanctuary in Mizoram, Gumti Wildlife Explanation: Law schools across India launched Sanctuary is famous wildlife reserve in Tripura. a collaborative legal aid clinic to help people Saramati is a peak rising above the surrounding excluded from the National Register of Citizens in peaks at the mountainous border of Nagaland Assam. state (India)

www.iasgatewayy.com

UPSC OCTOBER - 2019 DAILY CURRENT AFFAIRS CLASS TEST - 04 5. Consider the following Joint Exercises. a large gene family that encodes beta-lactamase 1. Nomadic Elephant - Indo-Mongolian enzymes called carbapenemases. Bacteria that Joint Exercise produce carbapenemases are often referred to in 2. Ekuverin - Indo Maldives Joint Exercise the news media as "superbugs" because infections 3. KAZIND - Indo-Kyrghstan Joint caused by them are difficult to treat. Exercise Which of the above pairs are correctly matched? 7. Recently, U.S. warned India that it may risk a) 1 and 2 only sanctions under CAATSA law. In this b) 1 and 3 only context consider the following statements c) 2 and 3 only regarding CAATSA law. d) All the above 1. CAATSA is a U.S. federal law through Answer: A which U.S. imposed sanctions on Iran, Explanation: The Joint Military Exercise and Russia. KAZIND-2019 between India and Kazakhstan has 2. CAATSA empowers the US President to started recently at Pithoragarh, Uttarakhand and impose at least five of the 12 listed continues till 15th October 2019. sanctions on persons engaged in a “significant transaction” with Russian 6. The term “NDLM-1” has been in news defence and intelligence sectors. recently. It refers to Which of the statements given above is/are a) New Delhi Landing Mechanism for VIP correct? Flights a) 1 only b) Protocol for dealing with stopping b) 2 only Elephant Deaths in case of c) Both 1 and 2 endotheliotropic herpesvirus deaths d) Neither 1 nor 2 c) An enzyme that makes bacteria resistant to Answer: C a broad range of antibiotics. Explanation: d) New Tele-medicine platform launched by Why this question? ISRO The U.S. officials have warned India that with the Answer: C decision to go ahead with the purchase of the Explanation: New Delhi metallo-beta- Russian S-400 missile defence system, India may lactamase 1 (NDM-1) is an enzyme that makes risk sanctions under CAATSA law. bacteria resistant to a broad range of beta-lactam What is CAATSA? antibiotics. These include the antibiotics of the Enacted in 2017, it is a US federal law that carbapenem family, which are a mainstay for the imposed sanctions on Iran, North Korea and treatment of antibiotic-resistant bacterial Russia. infections. The gene for NDM-1 is one member of www.iasgatewayy.com

UPSC OCTOBER - 2019 DAILY CURRENT AFFAIRS CLASS TEST - 04 Includes sanctions against countries that engage constructions built in ancient India in order to in significant transactions with Russia’s defence provide resting places for the wandering Buddhist and intelligence sectors. monks.

The Act empowers the US President to impose at 10. Consider the following statements about least five of the 12 listed sanctions on persons Shaheen Falcon engaged in a “significant transaction” with 1. It can live in a variety of habitats, ranging Russian defence and intelligence sectors. from the low altitude, arid landscapes, to the upper slopes of the Nilgiris. 8. Which one of the following Schedules of the 2. They are classified as “vulnerable” in the Constitution of India contains provisions International Union for Conservation of regarding anti-defection? Nature (IUCN) Red List of Threatened a) Second Schedule Species. b) Fifth Schedule Which of the above statement(s) is/are incorrect? c) Eighth Schedule a) 1 only d) Tenth Schedule b) 2 only Answer: D c) Both 1 and 2 Explanation: 10th Schedule – Anti – deflection d) Neither 1 nor 2 law. Answer: D Explanation: Both Statements are correct 9. Some Buddhist rock-cut caves are called

Chaityas, while the others are called Viharas. What is the difference between the two? a) Vihara is a place of worship, while Chaitya is the dwelling place of the monks b) Chaitya is a place of worship, while Vihara is the dwelling place of the monks c) Chaitya is the stupa at the far end of the cave, while Vihara is the hall axial to it d) There is no material difference between the two Answer: B Explanation: Chaityas refer to the halls enclosing the stupas. Chaityas were probably constructed to hold large numbers of devotees for prayer. Viharas on the other hand are

www.iasgatewayy.com

UPSC OCTOBER - 2019 DAILY CURRENT AFFAIRS CLASS TEST - 05 the murder of Mohammed Akhlaq – a Muslim 1. Recently PRAKASH portal was launched by resident of Dadri, Uttar Pradesh – by a mob a) Ministry of Coal & Power. claiming that he had eaten beef. b) Ministry of New and Renewable Energy

c) Meity 4. Consider the following statements about d) Niti Aayog Asian Development Bank Answer: A 1. The Asian Development Bank (ADB) is a Explanation: Power Rail Koyla Availability regional development bank established through Supply Harmony (PRAKASH) portal was in 1966. launched by the Ministry of Coal & Power. 2. ADB now has 67 members, of which 48 2. Sarbat Da Bhala Express was recently are from within Asia and the Pacific and flagged off from New Delhi. It forms the 19 outside. essence of teachings of 3. Currently, Japan and United States hold a) Guru Nanak Dev the largest proportion of shares. b) Acharya Vinoba Bhave 4. ADB provides financing to both c) Kabir Sovereign Nations and private d) Lal Bahadur Shastri companies. Answer: A Which of the above statements are correct?

a) 1,2,3 only 3. Consider the following statements about b) 1,3,4 only "Halt the Hate" c) 1 and 4 only 1. Halt the Hate’ is an interactive website d) All the above Answer: D that documents report of alleged hate Explanation: Factual statements about ADB crimes in India

2. It is an initiative of Journalists sans 5. NH 766 recently seen in news passes Borders through which of the following States Which of the above statement(s) is/are a) Kerala and Karnataka incorrect? b) Karnataka and Telangana a) 1 only c) Kerala and Tamil Nadu b) 2 only d) Andhra Pradesh and Tamil Nadu c) Both 1 and 2 Answer: A d) Neither 1 nor 2

Answer: B

Explanation: Amnesty’s Halt the Hate website was launched in September 2015 in the wake of

www.iasgatewayy.com

UPSC OCTOBER - 2019 DAILY CURRENT AFFAIRS CLASS TEST - 05 6. Consider the following statements with 7. Consider the following statements with respect to Minimum Alternate Tax: respect to Exchange Traded Funds (ETFs): 1. Minimum Alternate Tax is a direct tax. 1. ETFs are traded in Stock Markets. 2. It is imposed on book profit of 2. They are index funds. companies. 3. They are less liquid than Mutual Funds. 3. MAT is applicable to “Zero tax Which of the given statement/s is/are correct? companies”. a) 1 and 2 only Which of the given statement/s is/are b) 2 and 3 only Incorrect? c) 1 and 3 only a) 1 only d) 1, 2 and 3 b) 1 and 2 only Answer: A c) 2 and 3 only Explanation: ETFs hold a portfolio of securities d) 3 only that replicates an index and are listed and traded Answer: A on the stock exchange. Explanation: The Minimum Alternative Tax Since they trade like stocks and on stock (MAT) is imposed on book profit of companies exchanges, ETFs tend to be more liquid than who record nil or negligible profit to pay the usual mutual funds. Exchange Traded Funds (ETFs) are corporate income tax. essentially index funds. They track indexes like Book profit means the net profit as shown in the the Nifty, Sense. profit & loss account. MAT is an indirect tax. The objective of the 8. Consider the following statements about introduction of MAT is to bring into the tax net Cotton 4 Countries “zero tax companies” which in spite of having 1. It relates to talks in the WTO on cotton earned substantial book profits and having paid 2. India has been leading the group's handsome dividends, do not pay any tax due to negotiations various tax concessions and incentives provided Which of the above statement(s) is/are incorrect? under the Income-tax Law. a) 1 only Zero tax companies: The Income Tax Act allows b) 2 only companies to claim certain exemptions, deduct c) Both 1 and 2 certain expenses and make various provisions to d) Neither 1 nor 2 while calculating taxable income. Because of these Answer: B exemptions and deductions, the taxable profit Explanation: India is not even part of the group may become zero despite having substantial book profit. These companies are popularly known as Zero Tax companies.

www.iasgatewayy.com

UPSC OCTOBER - 2019 DAILY CURRENT AFFAIRS CLASS TEST - 05 9. In compliance with the Solid Waste Management Rules, 2016, India's first e- waste clinic is to come up in a) Delhi b) Bhopal c) Chennai d) Trivandrum Answer: B Explanation: The Bhopal Municipal Corporation (BMC) and the Central Pollution Control Board (CPCB) have joined hands to set up the country’s first e- waste clinic in Bhopal. CPCB will offer technical support.

10. The distribution of powers between the Centre and the States in the Indian Constitution is based on the scheme provided in the a) Morley-Minto Reforms, 1909 b) Montagu-Chelmsford Act, 1919 c) Government of India Act, 1935 d) Indian Independence Act, 1947 Explanation: Distribution of power between the Centre and the States in the Indian Constitution is based on the Government of India Act. 1935.

www.iasgatewayy.com

UPSC OCTOBER - 2019 DAILY CURRENT AFFAIRS CLASS TEST - 07 1. Consider the following statements with Answer: C respect to Youth Co:Lab Explanation: The Reserve Bank of India 1. It was Co-created by UNDP and the Citi releases its annual study of state-level budgets. Foundation 2. The initiative aims to create an enabling 3. Recently seen in news - ‘CyPAD’ refers to: ecosystem to promote youth leadership, a) Traces of Human Skeleton found in innovation, and social entrepreneurship Australia 3. India is the first country in Asia – Pacific b) Gene editing tool c) Cyber Crime Unit of Delhi Police region under the initiative d) Exoplanet in Habitable region Which of the above statement(s) is/are Answer: C Incorrect? Explanation: The Cyber Prevention, Awareness a) 1 only & Detection Centre (CyPAD) of Delhi Police and b) 1 and 2 only National Cyber Forensic Lab, MHA c) 3 only d) 1 and 3 only Answer: C 4. Consider the following about usage of Explanation: Co-created in 2017 by UNDP and Drones to supply Medicals the Citi Foundation, and operational in 25 1. The Maharashtra government has countries across the Asia Pacific region, the Youth adopted a framework to use drones for Co:Lab initiative aims to create an enabling last-mile delivery of essential medicals. ecosystem to promote youth leadership, 2. The project is a part of the World innovation, and social entrepreneurship. Economic Forum’s “Medicine from the 2. Consider the following about “The annual Sky” initiative study of state-level budgets” Which of the above statement(s) is/are correct? 1. It is released by Finance Commission a) 1 only 2. Since 2014-15, states have increasingly b) 2 only borrowed money from the market c) Both 1 and 2 3. During the last 5 years, except 2016-17, d) Neither 1 nor 2 state governments have regularly met Answer: B their fiscal deficit target of 3% of GDP Explanation: The Telangana government has Which of the above statement(s) is/are Correct? adopted a framework to use drones for last-mile a) 1 only delivery of essential medicals. b) 1 and 2 only c) 2 and 3 only d) All the above

www.iasgatewayy.com

UPSC OCTOBER - 2019 DAILY CURRENT AFFAIRS CLASS TEST - 07

5. The power to increase the number of judges c) GST Network in the Supreme Court of India is vested in d) None of the Above Answer: B a) The President of India Explanation: The Central Board of Direct b) The Parliament Taxation (CBDT) has unveiled a new initiative for c) The Chief Justice of India faceless e-assessment to impart greater efficiency, d) The Law Commission transparency and accountability in the Answer: B assessment process. Explanation: It is the Parliament which has the power to increase the number of judges in the 8. “Planet Nine” seen sometimes in news refers Supreme Court of India. Parliament increased the to : number of judges from the original eight in 1950 a) A hypothetical planet in the outer region of to eleven in 1956, fourteen in 1960, eighteen in the Solar System 1978, twenty-six in 1986 and thirty-one in 2008.

b) Goldilock Zone outside the Solar System

c) An Asteroid where traces of water was 6. The terms ‘WannaCry, Petya and Eternal found Blue’ sometimes mentioned in the news d) None of the above recently are related to Answer: A a) Exoplanets Explanation: Factual Statement b) Cryptocurrency c) Cyber-attacks 9. The MOSAiC mission is related to d) Mini satellites a) India’s mapping of languages which have Answer: C lost their written script in last 10 years Explanation: The Petya and WannaCry cyber- b) Study of Arctic Climate using a Drifting attacks in May and June 2017; are two of the Observatory biggest in history and impacted the finances of c) NASA’s mission to map Exo-planets companies throughout the globe. d) JAXA’s mission to land on Moon’s North Wanna Cry, which affected numerous Pole organizations, including the NHS, spread to 150 Answer: B countries and is estimated to have cost the global Explanation: The Multidisciplinary drifting economy £6bn. Observatory for the Study of Arctic Climate

expedition is a one-year-long expedition into the

7. Recently the National e-Assessment Centre Central Arctic, planned to take place from 2019 to (NeAC) was inagurated . It’s an initative of 2020. For the first time a modern research a) Reserve Bank of India icebreaker will operate in the direct vicinity of the b) Central Board of Direct Taxation North Pole year-round, including the nearly half

www.iasgatewayy.com

UPSC OCTOBER - 2019 DAILY CURRENT AFFAIRS CLASS TEST - 07 year long polar night during winter. In terms of The President has recently commuted death the logistical challenges involved, the total sentences to life imprisonment in at least 20 cases number of participants, the number of over the past nine years, based on the participating countries, and the available budget, recommendations received from the Ministry of MOSAiC represents the largest Arctic expedition Home Affairs (MHA). in history.

10. With reference to the Pardoning Powers of President, consider the following statements: 1. The power of pardon shall be exercised by the President on the advice of Council of Ministers. 2. Constitution does not provide for any mechanism to question the legality of decisions of President or governors exercising mercy jurisdiction. Which of the statements given above is /are correct? a) 1 only b) 2 only c) Both 1 and 2 d) Neither 1 nor 2 Answer: C Explanation: Both statements are correct. President cannot use suo moto Pardoning power there need to be advice by council of ministers.

But further, SC in Epuru Sudhakar case has given a small window for judicial review of the pardon powers of President and governors for the purpose of ruling out any arbitrariness. Why this Question:

www.iasgatewayy.com

UPSC OCTOBER - 2019 DAILY CURRENT AFFAIRS CLASS TEST - 09 1. Consider the following statements: 2. Consider the following statements regarding 1. Cotton is a Rabi crop which requires at Statue of Unity: least 210 frost free days in a year. 1. 'Loha Campaign' was initiated to collect 2. World Cotton Day is observed every year Metal scrap all over the country for the on October 7. construction of this Statue. Which of the statements given above is/are 2. The Sardar Vallabhbhai Patel Rashtriya correct? Ekta Trust (SVPRET) is responsible for a) 1 only planning and development of the complete b) 2 only project. c) Both 1 and 2 Which of the statements given above is/are d) Neither 1 nor 2 correct? Answer: B a) 1 only Explanation: Cotton is a kharif crop which b) 2 only requires at least 210 frost free days in a year. c) Both 1 and 2 Hence, statement 1 is not correct. d) Neither 1 nor 2 Answer: C The World Trade Organisation (WTO) is hosting Explanation: Metal scrap (mainly agriculture the event at the request of Cotton-4 (Benin, equipment scrap) was collected from the 6 lakh Burkina Faso, Chad and Mali) to celebrate their villages under the 'Loha Campaign' from across official application for recognition of October 7 as the country. So, statement 1 is correct. World Cotton Day by the United Nations. Hence, statement 2 is correct. The Sardar Vallabhbhai Patel Rashtriya Ekta Trust (SVPRET) has been created to manage the The World Trade Organisation (WTO) is resources and execution process of the project. organizing World Cotton Day event in This Apex Body is responsible for planning and collaboration with the United Nations Food and development of the project. So, statement 2 is also Agriculture Organization (FAO), the United correct. Nations Conference on Trade and Development (UNCTAD), the International Trade Centre (ITC) 3. Consider the following statements regarding and the International Cotton Advisory Committee “Nobel Prize” (ICAC). 1. The awardee for the Nobel Prize in the field of Peace was selected by a Committee elected by the Sweden Parliament. 2. One cannot nominate himself/herself for a Nobel Prize.

www.iasgatewayy.com

UPSC OCTOBER - 2019 DAILY CURRENT AFFAIRS CLASS TEST - 09 Which of the statements given above is/are next burning, and help authorities to act in correct? advance. a) 1 only b) 2 only Using the data, the Centre for Development of c) Both 1 and 2 Advanced Computing (C-DAC), under the aegis of d) Neither 1 nor 2 the Central Pollution Control Board, creates Answer: B probability maps to alert government agencies Explanation: The awardee for the Nobel Prize in about areas where the chances of stubble burning the field of Peace was selected by a 5-member is going to be high. Committee elected by the Norwegian Parliament. So, statement 1 is incorrect. The system can also track pollution load from stubble burning in places neighbouring the It has to be noted that except for the field of Peace, national capital, using satellite data. It can predict the awardees are selected by Sweden based the air pollution level for next 72 hours. It can also institutions. forecast the level of pollutants like particulate matter (PM) 2.5, PM10, and dust, coming from One cannot nominate himself/herself for a Nobel sources other than stubble burning. Prize. So, statement 2 is correct. 5. Consider the following statements about 4. Recently, an advanced Air Quality Early Ganga Aamantran Abhiyan: Warning System was launched, which can 1. It is a unique open-water rafting and predict places neighbouring Delhi that kayaking expedition on the Ganga River. are likely to burn crop residue on a given day. 2. The expedition consists of members It was launched by: from Indian Armed Forces, NDRF, a) Delhi Government Wildlife Institute of India as well as b) Ministry of Environment, Forest and CSIR-Indian Institute of Toxicology Climate Change. Research. c) Union Ministry of Earth Sciences Which of the statement(s) given above is/are d) Indian Meteorological Department correct? Answer: C a) 1 only Explanation: The system has been developed by b) 2 only Indian Institute of Tropical Meteorology, Pune, c) Both 1 and 2 under MoES. d) Neither 1 nor 2

Answer: C It uses data of stubble burning incidents from the past 15 years to predict the date and place of the

www.iasgatewayy.com

UPSC OCTOBER - 2019 DAILY CURRENT AFFAIRS CLASS TEST - 09 Explanation: Jal Shakti Ministry has Physics, and Physiology or Medicine by launched Ganga Aamantran Abhiyan, a unique The Nobel Foundation. open-water rafting and kayaking  It doesn’t include any specific field called expedition on the Ganga River to create Space and Psychology awareness on Ganga rejuvenation and water conservation. 7. Which of the following region is considered  The expedition team consists of members as the Third Pole (after the North and South from Indian Armed forces, NDRF, WWI and Poles) of the Earth? CSIR-IITR, a) The Siberian region  Apart from the awareness campaign, the team b) from CSIR–Indian Institute of Toxicology c) Hind-Kush-Himalayan Region Research will collect water samples from d) The Mariana Trench across diverse ranges of the river for the Answer: C purpose of water testing, while members of the Explanation: Wildlife Institute of India will undertake flora  The HKH region is considered the Third Pole and fauna census for the year 2019. [after the North and South Poles] and has significant implications for climate. 6. The Nobel Prizes are a set of recognition  The Third Pole, which contains vast given to fields of: cryospheric zones, is also the world’s largest 1. Literature store of snow and ice outside the polar region, 2. Space and the source of 10 major rivers, and, 3. Physics therefore, particularly sensitive to climate 4. Chemistry change. 5. Psychology  The Hindu-Kush-Himalayan (HKH) region 6. Physiology spans Afghanistan, Bangladesh, Bhutan, Select the correct answer using the codes given China, India, Kyrgyzstan, Mongolia, below: Myanmar, Nepal, Pakistan, Tajikistan, and a) 1, 3, 4 and 6 Uzbekistan. b) 3 and 4 only 8. ‘Bunad’ recently seen in news refers to: c) 1, 3 and 4 only a) Traditional folk costume of Norway d) 1, 2, 3 and 4 b) Traditional saree embroidery from Answer: A Rajasthan Explanation: c) A newly discovered Near Earth Asteroid  The Nobel Prizes are a set of recognition given d) None of the above to fields of Chemistry, Literature, Peace, Answer: A

www.iasgatewayy.com

UPSC OCTOBER - 2019 DAILY CURRENT AFFAIRS CLASS TEST - 09 Explanation: Explanation:  Bunad is Norway's traditional folk costume.  Putting in place a speedier approval  When Norwegian Ambassador Hans Jacob mechanism, Competition Commission has Frydenlund went to Rashtrapati Bhavan to introduced a green channel route for clearing present his credentials to President Ram Nath certain categories of mergers and acquisitions. Kovind recently, he was wearing a “bunad.”  Mergers and Acquisitions (M&As) or  Bunad is not a single kind of costume but an combinations beyond a certain threshold are umbrella term with several regional required to have mandatory approval from the variations. There are 400 different variations fair trade regulator. that come in different styles for men and  The green channel is aimed to sustain women. and promote a speedy, transparent and  A bunad often includes an apron, a headdress, accountable review of combination and a scarf or shawl, and is embroidered and cases, strike a balance between embellished with buckles, ornaments, facilitation and enforcement functions, jewellery and at times, blades. create a culture of compliance and  Bunads are expensive and typically worn on support economic growth. festive occasions.  Under this process, the combination is deemed to have been approved upon filing the 9. Consider the following statements about notice in the prescribed format. ‘Green Channel Combination’ recently seen  It has nothing to do with Pollution and Waste in News: Management. 1. It is a combination of methods to channel environmentally harmful waste safely into 10. Recently which of the following states have the atmosphere. come up with a draft policy to deliver 2. It has been brought up by Ministry of Medical Essentials through Drones: Environment Forest and Climate Change a) Telangana in collaboration with UNEP b) Andhra Pradesh Which of the statement(s) given above is/are c) West Bengal correct? d) Kerala

a) 1 only Answer: A b) 2 only Explanation: c) Both 1 and 2  Telangana government has adopted a d) Neither 1 nor 2 framework to use drones for last-mile delivery Answer: D of essential medical supplies such as blood and medical samples in an effort to increase the

www.iasgatewayy.com

UPSC OCTOBER - 2019 DAILY CURRENT AFFAIRS CLASS TEST - 09 access to healthcare to communities across the state.  The framework has been co-designed by the World Economic Forum (WEF) and Apollo Hospitals Group Healthnet Global Limited.  The project is a part of the WEF’s “Medicine from the Sky” initiative that aims to develop source materials for policymakers and health systems to analyse the challenges that come with drone delivery, and to compare this model with other competing delivery models.

www.iasgatewayy.com

UPSCOCTOBER- 2019 DAILY CURRENT AFFAIRS CLASS TEST - 10 1. Consider the following statements with 2. Sri Lanka, Bangladesh and Thailand are respect to MOSAiC Expedition: among the nations that have joined the 1. MOSAiC is the largest ever Arctic IFC-IOR coastal radar chain network. expedition. Which of the given statement/s is/are correct? 2. The mission is aimed at studying the a) 1 only impact of climate change on the Arctic. b) 2 only Which of the given statement/s is/are correct? c) Both 1 and 2 a) 1 only d) Neither 1 nor 2 b) 2 only Answer: A c) Both 1 and 2 The IFC-IOR, established in December 2018 in d) Neither 1 nor 2 Gurugram, India, is a single point centre linking Answer: C all coastal radar chains in the region to generate Explanation: Multidisciplinary drifting a seamless real-time picture of India’s Observatory for the Study of Arctic Climate 7,500-kilometre-long coastline. (MOSAiC) expedition is the largest ever Arctic expedition in history and will be the first to In addition to Maldives, several countries in the conduct a study of this scale at the North Pole for Indian Ocean Region have joined the coastal an entire year. radar chain network including Mauritius, Sri Lanka and Seychelles. It is spearheaded by the Alfred Wegener Institute in Germany. Bangladesh is set to join IFC-IOR while

Multidisciplinary drifting Observatory for the discussions are being held with Thailand as well.

Study of Arctic Climate (MOSAiC) expedition will 3. Which of the following are the aims of help the researchers better understand the Nutrient Based subsidy scheme? impact of climate change and aid in improved 1. Balanced use of fertilizer. weather projections. 2. Improving the agricultural productivity. The mission is aimed at studying the impact of 3. Promoting growth of indigenous climate change on the Arctic and how it could fertilizer industry. affect the rest of the world. 4. Reducing burden of subsidy. Which of the given statement/s is/are correct? 2. Consider the following statements: a) 1 and 2 only 1. Information Fusion Centre – Indian b) 1, 2 and 3 only Ocean Region (IFC-IOR) is an c) 1,2 and 4 only information-sharing hub of maritime d) All the above data established in Gurugram. Answer: D

www. vetriias.com / www.iasgatewayy.com

UPSCOCTOBER- 2019 DAILY CURRENT AFFAIRS CLASS TEST - 10 Explanation: Nutrient Based Subsidy (NBS) 2. It is a 6-member committee with RBI programme for fertilizer was initiated in the year governor as its chairman and Finance 2010. Under the scheme, a fixed amount of secretary as its ex-officio member. subsidy decided on an annual basis is provided Which of the given statement/s is/are correct? on, based on the nutrient content present in a) 1 only them. The scheme is being implemented by the b) 2 only Department of Fertilizers. c) Both 1 and 2 d) Neither 1 nor 2 In India, urea is the only controlled fertilizer and Answer: A is sold at a statutory notified uniform sale price. Explanation: Monetary Policy Committee The scheme allows the manufacturers, (MPC) is a 6-member committee formed after marketers, and importers to fix the MRP each the amendment in the RBI Act, 1934 through the grade of subsidized Phosphatic and Potassic Finance Act, 2016. The basic objective of MPC is (P&K) fertilizers, except for Urea of the to maintain price stability and accelerate the Phosphatic and Potassic fertilizers at reasonable growth rate of the economy. levels. Monetary policy refers to the policy of the The scheme aims at ensuring that sufficient Reserve Bank of India with regard to the use of quantity of P&K is at the farmer’s disposal at monetary instruments under its control to statutory controlled prices. So that the achieve the goals of GDP growth and lower agricultural growth can be sustained and inflation rate. The RBI is authorized to made balanced nutrient application to the soil can be monetary policy under the Reserve Bank of India ensured. Act, 1934.

The Monetary Policy Committee determines the It aims at ensuring balanced use of fertilizers, policy rates required to achieve the inflation improving the agricultural productivity, target. promoting the growth of the indigenous fertilizers industry and also reducing the burden As per the provisions of the RBI Act, out of the of Subsidy. six Members of Monetary Policy Committee,

three Members will be from the RBI and the 4. Consider the statements regarding other three Members of MPC will be appointed Monetary Policy Committee (MPC): by the Central Government.

1. It was formed in 2016 to maintain price stability and accelerate the growth rate

of the economy.

www. vetriias.com / www.iasgatewayy.com

UPSCOCTOBER- 2019 DAILY CURRENT AFFAIRS CLASS TEST - 10 5. Nandankanan Zoological Park, which was Explanation: Zero Budget Natural Farming recently in news is located in the state of (ZBNF) is a method of chemical-free agriculture a) Madhya Pradesh drawing from traditional Indian practices.

b) Odisha It was originally promoted by Maharashtrian c) Bihar agriculturist and Padma Shri recipient Subhash d) Uttarakhand Palekar, who developed it in the mid-1990s as an Answer: B alternative to the Green Revolution’s methods

Explanation: Four calves between the ages of driven by chemical fertilizers and pesticides and six and 10 have died in NandanKanan Zoo in intensive irrigation. Bhubaneswar, Odisha. The four deaths in NandanKanan Zoo are the first reported cases of The cost of production in ZBNF is zero as EEHV-related deaths in an Indian zoo. farmers don’t require to buy any inputs.

EEHVs or elephant endotheliotropic herpes virus The farmers practicing ZBNF either of small land as a type of herpes virus that can cause a highly holding or large land holding prepare the low- fatal haemorrhagic disease in young Asian cost cow urine and dung-based formulation on elephants. farm by procuring the required inputs locally from the village.

The disease is usually fatal, with a short course of 28-35 hours. There is no true cure for herpes 7. The Report ‘Global Competitiveness Index’ viruses in animals or in humans. was released by

6. Which of the following is correct regarding a) World Bank Zero Budget Natural Farming? b) International Monetary Fund 1. It is a new agriculture technology c) World Economic Forum developed by ICAR. d) OECD 2. It aims to reduce the adverse impact of Answer: C the green revolution. 3. The input cost of the farmer’s will be

reduced.

Select the correct codes:

a) 3 only

b) 1 and 3 only

c) 1,2 and 3

d) 2 and 3 only Answer: D

www. vetriias.com / www.iasgatewayy.com

UPSCOCTOBER- 2019 DAILY CURRENT AFFAIRS CLASS TEST - 10 Explanation: The Global Competitiveness Index (GCI), which was launched in 1979, maps The batteries have a high energy density the competitiveness landscape of 141 economies (Energy density is the amount of energy stored in through 103 indicators organised into 12 pillars. a given system or region of space per unit Twelve Pillars: volume). 1. Institutions 7. Product market Lithium –ion battery is a lightweight, 2. Infrastructure 8. Labour market hardwearing battery that could be charged 3. ICT adoption 9. The financial system hundreds of times before its performance deteriorated. 4. Macroeconomic stability 10. Market size The advantage of lithium-ion batteries is that 5. Health 11. Business dynamism they are not based upon chemical reactions that 6. Skills 12. Innovation capability break down the electrodes, but upon lithium ions flowing back and forth between the anode and

cathode. GCI 2019 has been released. Compared to last year India has moved down 10 places to rank 9. Consider the following statements: 68th. India was ranked 58th last year. 1. 51 Pegasi was the first exoplanet It is among the worst-performing BRICS nations discovered orbiting a sun like star. along with Brazil (which ranked 71st). Singapore 2. Cosmic microwave background has replaced the U.S. as the world’s most radiation is an evidence of expanding competitive economy. universe.

8. Consider the Lithium-ion batteries: Which of the following statements is /are 1. They are rechargeable batteries with correct? high energy densities. a) 1 only 2. They are significantly lighter and are b) 2 only widely used in the portable electronics. c) Both 1 and 2 Which of the given statement/s is/are correct? d) Neither 1 nor 2 a) 1 only Answer: C b) 2 only Explanation: The Swiss astronomers have c) Both 1 and 2 been awarded the Nobel for their discovery of d) Neither 1 and 2 the first ever exoplanet orbiting a star. Answer: C Exoplanet 51 Pegasi was the 1st exoplanet Explanation: Lithium-ion batteries have discovered orbiting a sun-like star, about revolutionised our lives and are used in 50light years away from earth in the everything from mobile phones to laptops and constellation of Pegasus. So, statement 1 is electric vehicles. correct.

www. vetriias.com / www.iasgatewayy.com

UPSCOCTOBER- 2019 DAILY CURRENT AFFAIRS CLASS TEST - 10 Answer: A Immediately after the big bang, the universe Explanation: Statement 3 is wrong was so hot that the thermonuclear reactions Satnami sect is thought to be an offshoot of (that areusually seen in stars today) happened Ravidasi sect and comprised of lower strata of everywhere in the universe leading to the Hindu society, particularly, leather workers, formation of primodialelements, hydrogen and sweepers, carpenters, Goldsmiths etc. helium. Peebles (NOBEL Prize winner) predicted that this thermonuclear fusion of Who are Satnamis? hydrogen into helium atoms led to release of Originally, they were a militant sect of Hindu high-energy shortwave photons which is Worshippers. known to be cosmic background radiation. As Founded by a saint named “Birbhan” in 1657 in the universe expanded this radiation also Narnaul in Haryana. expanded becoming long-wave (microwave) which is why it is called cosmic microwave The major religious activity of this sect is to background radiation which fills the entire chant and meditate the true names (Sat-Nam) of space. God, specially Rama and Krishna.

Thus, CMB is an evidence for expansion of The followers of this sect kept their heads shaven universe. So, statement 2 is also correct.

(thus called Mundiyas) and abstained from 10. With reference to Satnami rebellion, liquor and meat. consider the following statements? The religious granth of the Satnamis is 1. Originally, Satnami's were a militant called Pothi. sect of Hindu Worshippers. 2. The major religious activity of Satnami sect is to chant and meditate the true names (Sat-Nam) of God, specially Rama and Krishna. 3. Satnami sect is thought to be an offshoot of vedanta and comprised of elites of the Hindu society. Which of the statements given above is/are correct? a) 1 and 2 only b) 2 and 3 only c) 3 only d) 1, 2 and 3

www. vetriias.com / www.iasgatewayy.com

UPSC OCTOBER- 2019 DAILY CURRENT AFFAIRS CLASS TEST - 11

1. Consider the statement regarding Graded 2. Which of the following State has launched Response Action Plan the first-ever public information portal in 1. It is aimed to measure air quality based India? on PM 2.5 and PM 10 in the atmosphere. a) Tamil Nadu 2. It will be enforced by Central Pollution b) Rajasthan Control Board. c) Kerala 3. Based on air quality, grades are classified d) Odisha in 6 categories. Answer: B Which of the statement(s) given above is/are Explanation: The first-ever public information correct? portal (Jan Soochna Portal) was launched in a) 1 only Rajasthan by promising to provide information b) 1 and 2 only about government authorities and departments c) 1 and 3 only suo moto to the public in the Right to Information d) None of the Above Act. Answer: A Explanation: Graded Response Action Plan Section 4(2) of the RTI Act mandating the public defines the measures to taken based on-air quality authorities to disclose information in the public on the basis of PM 2.5 and PM 10 in the domain, so that the people need not file atmosphere. applications under the law to obtain information.

Based on the air quality the grades have been The State government has collaborated with the classified as Emergency, Severe, very poor and civil society groups to develop the portal, the first Moderate poor. of its kind in the country, initially giving information pertaining to 13 departments on a It will be enforced by Environment Pollution single platform. Control Authority (EPCA). Jan Soochna Portal launched under the Rajasthan Under this plan emergency measures will be Innovation Vision (RAJIV) campaign. automatically enforced in NCR if level of PM2.5 3. Which state has granted infrastructure breaches 300 micrograms per cubic metre project status for a $10-billion plan to build (µgm/m3) and PM10 levels stay above 500 the world’s first ultra-fast hyper loop (µgm/m3) for two consecutive days. project?

a) Maharashtra The plan recommends measures like odd-even car b) Tamil Nadu rationing scheme and ban on construction c) Uttar Pradesh activities to combat air pollution. d) Gujarat www.iasgatewayy.com

UPSC OCTOBER- 2019 DAILY CURRENT AFFAIRS CLASS TEST - 11 Answer: A  Nobel Prize in Physiology or Medicine: The Explanation: The very basis of Hyper loop is to Karolinska Institute reduce the friction against the air or the ground,  Nobel Prize in Literature: The Swedish which restricts the speed of conventional trains. Academy Hyper loop involves a long vacuum tube with pods  Nobel Peace Prize: A five-member carrying passengers beamed through it. Committee elected by the Norwegian The Maharashtra government accorded the Parliament infrastructure status to the Mumbai-Pune ultra- fast hyper loop transport project that seeks to 5. Which of the following international reports reduce the travel time between the two cities to are published by World Economic Forum: just 23 minutes. 1. Global Gender Gap Report

4. Consider the following institutes and the 2. Future of Jobs Report Nobel Prizes awarded by them: 3. World Economic Outlook Institute Nobel Prize Select the correct answer using the code given 1.Royal Swedish below: Academy of Sciences - Physics, chemistry a) 1 only and Economics b) 1 and 2 only 2.Karolinska Institute - Medicine c) 1 and 3 only 3.Swedish Academy - Literature d) 1,2 and 3 4.Norwegian Nobel Answer: B Committee - Peace Explanation: World Economic Outlook is Which of the pairs given above is/are correctly published by IMF, whereas the other two reports matched? are by World Economic Forum.

a) 1 only 6. Consider the following statements relating b) 1 and 2 only to GEMINI system: c) 1,2 and 3 only 1. GEMINI is a portable device that is d) 1,2,3 and 4 devised to help fishermen to access Answer: D warnings and alerts even when they are Explanation: The institutions that choose winners: The Nobel Committees of the prize- outside the signal range of their phone awarding institutions are responsible for the companies. selection of the candidates, the institutions being: 2. This device has been developed by ISRO  Nobel Prize in Physics, Nobel Prize in and works on the GAGAN platform. Chemistry, Prize in Economic Sciences: The Which of the statement(s) given above is/are Royal Swedish Academy of Sciences correct?

www.iasgatewayy.com

UPSC OCTOBER- 2019 DAILY CURRENT AFFAIRS CLASS TEST - 11 a) 1 only reduce cost of fertilizer application. Not a subsidy b) 2 only scheme.

c) Both 1 and 2 All other schemes are subsidised based

d) Neither 1 nor 2 Answer: A Bhavantar Bhugtan Yojana in Madhya Pradesh Explanation: To avoid communication was sought to provide relief to farmers by blackouts that led to 20 fishermen going missing providing the differential between MSPs and in the aftermath of Cyclone Okchi in 2017, a slew market prices. of government departments, research agencies The Rythu Bandhu scheme of the Telangana and private companies have developed GEMINI, provides ₹4,000 per acre for every season to all a portable receiver linked to ISRO-satellites, that the farmers of the state. Similar initiatives have is “fail-proof” and warn fishermen of danger. So, also been framed in Jharkhand and Odisha. statement 1 is correct. Pradhan Mantri Kisan Samman Nidhi: This device has been devised by The Indian Under this programme, vulnerable landholding National Centre for Ocean Information Services farmer families, having cultivable land upto 2 (INCOIS), in collaboration with Accord, a private hectares, will be provided direct income support company. GEMINI works on GAGAN, an India- at the rate of Rs. 6,000 per year. made global positioning system and relies on the positioning system by ISRO’s GSAT satellites. This income support will be transferred directly So, statement 2 is incorrect. into the bank accounts of beneficiary farmers, in three equal instalments of Rs. 2,000 each. 7. With reference to Agriculture subsidies, which of the following is NOT an Agriculture 8. The word ‘triple billion target’ sometimes subsidy scheme. seen in news is: a) Pradhan Mantri Kisan Samman Nidhi a) 5-year strategic plan launched by WHO. b) Bhavantar Bhugtan Yojana in Madhya b) Target set be RBI to add up to 3 billion Pradesh dollar to its reserves till 2020. c) Neem coated urea scheme. c) An ambitious plan started by Ministry of d) The Rythu Bandhu scheme of the health to reduce infant mortality across the Telangana country. d) None of the above Answer: C Answer- A Explanation: Neem coated Urea Explanation: WHO ‘Triple Billion’ targets: This scheme is initiated to regulate use of urea, It is a strategic plan for the next five years. enhance availability of nitrogen to the crop and

www.iasgatewayy.com

UPSC OCTOBER- 2019 DAILY CURRENT AFFAIRS CLASS TEST - 11 One billion more people to be benefitted from Under the scheme there will be zero expense Universal Health Coverage (UHC) access to identification and management of complications during and after the pregnancy.

One billion more people to be protected from health emergencies The government will also provide free transport from home to health institutions. One billion more people to be covered for better health and well-being. 10. Consider the following statements about

World Mental Health Day: 9. Recently Government of India has launched 1. The theme for 2019 was ‘Mental Health a scheme called SUMAN, this scheme aims Promotion and Suicide Prevention.’ to: 2. It is observed every year on 10th a) Distribute free sanitary pads October. b) Provide free electricity to farmers Which of the statement(s) given above is/are c) Bring down maternal and infant mortality correct? rates a) 1 only d) Empower women by encouraging training b) 2 only in martial arts c) Both 1 and 2 Answer: C d) Neither 1 nor 2 Explanation: Surakshit Matritva Aashwasan Answer: C (SUMAN) Scheme was recently launched by Explanation: Both statements are correct. Union Health Ministry.

Under the scheme, the beneficiaries visiting public health facilities are entitled to several free services.

These include at least four ante natal check-ups that also includes one check-up during the 1st trimester, at least one check-up under Pradhan Mantri Surakshit Matritva Abhiyan, Iron Folic Acid supplementation, Tetanus diptheria injection and other components of comprehensive ANC package and six home-based new born care visits.

www.iasgatewayy.com

UPSC OCTOBER- 2019 DAILY CURRENT AFFAIRS CLASS TEST - 14

1. Consider the following with respect to Why this Question: Agatti Island: To better gauge the impact of climate 1. It belong to Andaman and Nicobar change on the Hindu Kush mountains, which island groups. includes the Himalayas, and spruce up data- 2. It is made of coral reef. gathering, theIndia Meteorological Department (IMD) will collaborate with meteorological Which of the statements is/are incorrect? agencies in China and Pakistan, among others, to a) 1 only provide climate forecast services to countries in b) 2 only the region. c) Both 1 and 2

d) Neither 1 nor 2 Answer: A 3. Which of the following statements correctly explains the term ‘Tax Haven’? 2. With reference to ' Hindu kush a) A country which gives tax exemptions to Himalayan region ' consider following the foreign citizens that there will b) be no tax on investing the money in their statements. country 1. The Hindu Kush-Himalayan region c) Subsidy given by the government in taxes spans an area in Kazakhstan d) Tax evasion in the domestic country Afghanistan, Bangladesh, Bhutan, e) To impose equal taxes on domestic China, India, Myanmar, Nepal, and producers and foreign producers Pakistan. Answer: A 2. The region stores more snow and ice Explanation: Tax havens are countries that than anywhere else in the world outside have low or near-zero tax rates, especially for the polar regions, giving its name: ’The some kinds of transactions. Tax havens, which Third Pole‘. help rich corporations and businessmen, avoid Which of the statements given above is/are paying high taxes on their income. Singapore, correct? Hong Kong and Mauritius are the popular ones. a) 1 only Multinationals set up their holding companies in b) 2 only these locations which then invest in operations c) Both 1 and 2 located at other high-tax locations. So, even as d) Neither 1 nor 2 the company carries out its real business in a Answer: B high-tax regime such as the US or India, it’s able Explanation: Statement 1 is wrong to dodge the taxman by showing a large share of Hindu kush is not there in Kazakhstan, it is there profits as emanating from a tax haven. in Afghanistan, Bangladesh, Bhutan, China, Tax havens as “anti-poor” since they help the rich India, Myanmar, Nepal, and Pakistan. avoid paying taxes to governments. Several

www.iasgatewayy.com

UPSC OCTOBER- 2019 DAILY CURRENT AFFAIRS CLASS TEST - 14 governments have also come together to crack 5. Consider the following statements on down on tax havens, for only a collective effort Minimum Alternate Tax can help. 1. It is a tax introduced in the finance Act of 2016. 4. The term ‘Base Erosion and Profit Shifting’ 2. The objective of the tax is to check the is sometimes seen in the news in the tax avoidance. context of Which of the statements given above is/are a) World Bank correct? b) International Monetary Fund a) 1 only c) World Trade Organization b) 2 only d) Organisation of Economic Cooperation c) Both 1 and 2 and Development d) Neither 1 and 2 Answer: D Answer: B Explanation: Base Erosion and Profit Shifting Explanation: MAT is levied to facilitate the (BEPS) refers to those instances where gaps taxation of ‘zero tax companies’ i.e., those between different tax rules leads to tax avoidance companies which show zero or negligible income causing harm to the government. It refers to all to avoid tax. those artificial arrangements where: MAT is levied at the rate of 18.5% of the book Due to gaps in application of the bilateral tax profits. Now with the recent amendments, MAT treaties, cross border activities may go untaxed in is reduced to 15% for those who continue to avail any of the two countries. exemptions and incentives.

No or low tax is paid by shifting profits to low tax Minimum Alternate Tax (MAT) is a tax jurisdictions and shifting losses and high effectively introduced in India by the Finance Act expenditures to high tax jurisdictions. of 1987, vide Section 115J of the Income Tax Act, Originally, OECD had started the BEPS project in 1961 (IT Act). response to the 2008 financial crisis in order to create sustainable economic growth. 6. Consider the following statements: 1. Andaman and Nicobar Islands are It was formally launched in 2012 by the G-20 separated by eight-degree channel. Finance Ministers who in turn called on OECD to 2. Shompen tribes are part of the develop an action plan to address BEPS issues in Particularly Vulnerable Tribal Group a co-ordinated and comprehensive manner and (PVTG) and are native to Lakshadweep develop an action plan. Islands.

www.iasgatewayy.com

UPSC OCTOBER- 2019 DAILY CURRENT AFFAIRS CLASS TEST - 14 Choose the correct statement(s) using the code Indian Railway Catering and Tourism given below: Corporation Ltd. (IRCTC) is a Mini Ratna a) 1 only Category-I (conferred in 2008) Central Public b) 2 only Sector Enterprise. Hence, statement 2 is also c) Both 1 and 2 correct. d) Neither 1 nor 2 8. The proposed Green Wall of India stretches Answer: D from: Explanation: The Ten Degree Channel is a a) Kerala to Maharashtra channel that separates the Andaman Islands and b) Tamil Nadu to West Bengal Nicobar Islands from each other in the Bay of c) Gujarat to Delhi Bengal. So, statement 1 is incorrect. d) Meghalaya to Arunachal Pradesh

The Shompen or Shom Pen are the indigenous Answer: C people of the interior of Great Nicobar Island Explanation: The Central Government of India (and not Lakshadweep), part of the Indian union is going to take up an ambitious plan of creating territory of Andaman and Nicobar Islands. They a 1,400 km. long and 5 km wide green belt from are part of PVTG. So, statement 2 is also Gujarat to the Delhi-Haryana border, which will incorrect. be known as the Green Wall of India.

The plan is inspired by Africa’s ‘Great Green 7. Consider the following statements Wall’ project, running from Senegal (West) to regarding IRCTC: Djibouti (East), which came into effect in 2007.

1. IRCTC is under the administrative control of Ministry of Railways. The overarching objective of India’s Green Wall will be to address the rising rates of land 2. IRCTC is a Mini Ratna Category-I degradation and the eastward expansion of the central public sector enterprise. Thar desert. Choose the correct statement(s) using the code given below: The green belt being planned from Porbandar to a) 1 only Panipat will help in restoring degraded land b) 2 only through afforestation along the Aravali hill c) Both 1 and 2 range. It will also act as a barrier for dust coming d) Neither 1 nor 2 from the deserts in western India and Pakistan.

Answer: C 9. From West to East, which one of the Explanation: IRCTC is completely owned and following is the correct sequence of the controlled by the Ministry of Railways. Hence, tributaries of river Ganga? statement 1 is correct.

www.iasgatewayy.com

UPSC OCTOBER- 2019 DAILY CURRENT AFFAIRS CLASS TEST - 14 a) Kosi-Gandak-Ghaghara-Gomati Answer: C b) Gomati-Gandak-Ghaghara-Kosi Explanation: It will be a 1,400km long and c) Kosi-Ghaghara-Gandak-Gomati 5km wide green belt from Gujarat to the Delhi- d) Gomati-Ghaghara-Gandak-Kosi Haryana border, on the lines of the “Great Green Wall” running through the width of Africa, from Answer: D Dakar (Senegal) to Djibouti, to combat climate Explanation: Major left-bank tributaries of change and desertification. Ganga from West to East are Ram Ganga, Gomti,

Ghaghara, Gandak, Kosi, and Mahananda. If approved, this may turn out to be a legacy Hence, option (D) is the correct answer. programme in India’s efforts to deal with land

degradation and the eastward march of the Thar

desert.

India seeks replicate the idea as a national

priority under its goal to restore 26 million

hectares of degraded land by 2030.

The green belt may not be contiguous but would

roughly cover the entire degraded Aravali range

through a massive afforestation exercise. 10. The term ' Green wall ' sometimes

mentioned in the news recently are related Why this Question: to The Centre is mulling an ambitious plan to create a) a project by Highway authority of India of a green wall on North- Western part of India. planting trees on both sides of National

highways across India.

b) It is ambitious project by Central government to build green wall on borders of India Bangladesh similar to great wall of china. c) It is a 1,400km long and 5km wide green belt from Gujarat to the Delhi-Haryana border. d) to bulid a artificial green wall near Western Ghats to protect it from heavy rainfall.

www.iasgatewayy.com

UPSC OCTOBER- 2019 DAILY CURRENT AFFAIRS CLASS TEST - 15 1. With reference to solar power production in and for matters connected Electricity Tariff India, consider the following statements: regulation. 1. India is the third largest in the world in SECI has a power-trading license, but it does not the manufacture of silicon wafers used in set solar power tariffs. photovoltaic units.

2. The solar power tariffs are determined by 2. Consider the following statements regarding the Solar Energy Corporation of India. the Ujwal DISCOM Assurance Yojana Which of the statements given above is/are (UDAY): correct? 1. The motive of UDAY is to improve the a) 1 only b) 2 only health and performance of the c) Both 1 and 2 distribution companies. d) Neither 1 nor 2 2. The motive of UDAY is to improve the Answer: D health and performance of the Public Explanation: Not a single silicon chip for solar Sector Banks of the country. energy is manufactured in India. 3. One of the salient features of the scheme is to increase supply of domestic coal to Every solar panel which is made in India is substitute for imported coal. assembled while all the material comes from Which of the statements given above is/are China, Europe and some other countries. India is correct? yet to develop semiconductor clusters. a) Only 1

CERC determines solar power tariffs and b) 1 and 2 regulates the tariff of generating companies c) 1 and 3 owned or controlled by the Central Government. d) All of the above Central Electricity Regulatory Answer: C Commission (CERC), a key regulator of power Explanation: The motive of UDAY is to sector in India, is a statutory body functioning improve the health and performance of the with quasi-judicial status under Electricity Act power distribution companies. The salient 2003. features of the schemes are:

CERC was initially constituted on 24 July 1998 under the Ministry of Power’s Electricity States shall take over 75 per cent of discom debt Regulatory Commissions Act, 1998 for outstanding as of September 2015. rationalization of electricity tariffs transparent policies regarding subsidies, promotion of Reduction of Aggregate Technical & Commercial efficient and environmentally benign policies, (AT&C) losses to 15 per cent by 2018-19. Reduction in difference between average cost of www.iasgatewayy.com

UPSC OCTOBER- 2019 DAILY CURRENT AFFAIRS CLASS TEST - 15 supply and average revenue realized (ARR) by accounting or auditing system, improvement in 2018-19. billed energy based on metered consumption and improvement in collection efficiency. Increased supply of domestic coal to substitute for imported coal. States shall take over future 4. Consider the following statements losses of discoms in a phased manner. regarding Extinction Rebellion:

1. It is a pro-democracy movement in Banks/FIs not to advance short term debt to Hong-Kong. discoms for financing losses. 2. It is established by environmentalists to

act as a pressure group for government 3. Consider the following statements action. regarding the salient features of the Choose the correct statement using the code Integrated Power Development Scheme given below: (IPDS): a) 1 only 1. Strengthening of sub-transmission and b) 2 only distribution network in urban areas. c) 1 and 2 only 2. Metering of distribution transformers/feeders/consumers in rural d) Neither 1 nor 2 areas. Answer: B 3. IT enablement of distribution sector and Explanation: Extinction Rebellion (XR) is a strengthening of distribution network. civil society campaign group. It was established

Which of the following statement(s) is/are in 2018 in Britain by environmentalists and has correct? become one of the world's fastest-growing a) Only 1 environmental movements.

b) 1 and 2 So, statement 1 is incorrect and statement 2 is c) 1 and 3 correct. d) All of the above Answer: C Its objective is to use non-violent civil Explanation: Integrated Power Development disobedience to achieve radical change in order Scheme has replaced the UPA’s Restructured to minimise the risk of human extinction and Accelerated Power Development Programme. ecological collapse by averting climate The major focus of the scheme is to improve the breakdown and halt biodiversity loss. transmission and distribution network in urban areas of the country.

It also promises help in reduction of AT&C losses, establishment of IT enabled energy

www.iasgatewayy.com

UPSC OCTOBER- 2019 DAILY CURRENT AFFAIRS CLASS TEST - 15 5. Which of the following statement(s) is/are a) 1 and 3 only correct regarding Gossypol, which is b) 1,2 and 3 only recently in news: c) 1,3 and 4 only 1. It is a substance which is produced in d) 2,3 and 4 only the cotton seed. Answer: C 2. Gossypol is included in the cotton seed Explanation: Five rathas is also situated at through genetic modification to improve Mamallapuram.

plants ability to withstand diseases. 7. Consider the following statements on data Choose the correct statement using the code given below: localisation: a) 1 only 1. India does not have any rule as of now b) 2 only on Data localisation. c) Both 1 and 2 2. India has accepted all the clauses of e- d) Neither 1 nor 2 commerce chapter of RCEP agreement. Answer: A Which of the statements given above is/are Explanation: Cottonseed meal is a by-product correct: of cotton that is used for animal feeding because a) 1 only it is rich in oil and proteins. However, gossypol b) 2 only toxicity limits cottonseed use in animal feed. c) Both 1 and 2 d) Neither 1 nor 2 High concentrations of free gossypol may be Answer: D responsible for acute clinical signs of gossypol Explanation: RBI has issued norms regarding poisoning which include respiratory distress, data localisation. impaired body weight gain, anorexia, weakness, So, statement 1 is incorrect. apathy, and death after several days. So On the grounds of data localisation, India has statement 1 is correct and 2 is incorrect. disapproved the chapter of e-commerce and

didn’t sign that in the RCEP agreement. So, 6. Which of the following pairs are correctly statement 2 is also incorrect. matched?

1. Arjuana’s Penance - Mamallapuram 2. Five rathas - Badami 8. Which of the following is the main objective 3. Shore temple - Mamallapuram of mHariyali App? 4. Swarga Brahma temple - Alampur a) To encourage public engagement in planting trees and other such green drives Choose the correct answer using the code given b) To increase the participation of SHGs and below: NGOs in the policy making process

www.iasgatewayy.com

UPSC OCTOBER- 2019 DAILY CURRENT AFFAIRS CLASS TEST - 15 c) Encouraging public to save trees in forest  This is called the “elastocaloric” effect. areas  In the elastocaloric effect, the transfer of d) None of the above heat works much the same way as when

Answer: A fluid refrigerants are compressed and Explanation: mHariyali expanded.  The app is aimed to encourage Public  Researchers from multiple universities have engagement in planting trees and other such found that the elastocaloric effect, if Green drives. harnessed, may be able to do away with the  People can now upload information/photos need of fluid refrigerants used in fridges of any plantation done by them, which is and air-conditioners.

linked to app and will be displayed on the 10. Consider the following statements with website epgc.gov.in. respect to Pani Bachao Paisa Kamao Scheme  The App provides for automatic geo-tagging 1. It is a pilot project launched by the of plants. This app will also enable nodal Punjab State Power Corporation officers to periodically monitor the Limited (PSPCL). plantation. 2. The objective of the scheme is to  The App is user friendly and works on any encourage the farmers to save electricity android mobile phone. and reduce the use of tube wells and

ground water. 9. Which of the following is the example of Which of the statement(s) given above is/are Elastocaloric effect? correct? a) Cooling effect produced when rubbers a) 1 only bands are twisted and untwisted b) 2 only b) Storage of Calorie when nylon and c) Both 1 and 2 polyethylene are cooled d) Neither 1 nor 2 c) Heating effect produced when rubbers Answer: C bands are twisted and untwisted Explanation: The Punjab government has d) None of the above launched a pilot project last year 'Paani Bachao, Answer: A Paise Kamao' to check depletion of underground Explanation: When rubbers bands are twisted water. Under this scheme, farmers will get and untwisted, it produces a cooling effect. monetary benefit for less use of tube well.  When a rubber band is stretched, it absorbs The purpose of the scheme was to ensure that heat from its environment, and when it is farmers use the released, it gradually cools down. power with restraint to pump out water from earth.

www.iasgatewayy.com

UPSC OCTOBER- 2019 DAILY CURRENT AFFAIRS CLASS TEST - 16 2. Consider the following statements 1. Consider the following statements regarding L2Pro India: regarding Interconnect Usage Charge 1. This app has been launched to help (IUC):

Indian start-ups to know about the IPR 1. IUC is a cost paid by mobile telecom rights. operator in India to Telecom Regulatory 2. This app is launched by the Department Authority of India (TRAI). of Science and Technology. 2. The IUC charges are fixed by Telecom Choose the correct statement using the code Regulatory Authority of India (TRAI). given below: Choose the correct statement using the code a) 1 only given below: b) 2 only a) 1 only c) 1 and 2 only b) 2 only d) Neither 1 nor 2 c) 1 and 2 only Answer: A d) Neither 1 nor 2 Explanation: The modules of this e-learning Answer: B platform (L2Pro India) will aid and enable youth, Explanation: Interconnect Usage Charge or innovators, entrepreneurs and small and IUC is a cost paid by one mobile telecom medium industries (SMEs) in understanding operator to another, when its customers make IPRs for their ownership and protection, outgoing mobile calls to the other operator’s integrate IP into business models and obtain customers. value for their R&D efforts. So, statement 1 is incorrect. So, statement 1 is correct. The IUC charges are fixed by Telecom Regulatory Department for Promotion of Industry and Authority of India (TRAI). So, statement 2 is Internal Trade (DPIIT) has launched the website correct. and mobile application L2Pro India [Learn to 3. Consider the following statements Protect, Secure and Maximize Your Innovation] regarding ‘Nobel Prize for Economic on Intellectual Property Rights (IPRs).so Sciences- 2019’: statement 2 is incorrect. 1. The award for the year 2019 was given

to the laureates for their experimental

approach towards alleviation of global

poverty.

2. It was awarded to an Indian for the first

time. Which of the statements given above is/are correct?

www.iasgatewayy.com

UPSC OCTOBER- 2019 DAILY CURRENT AFFAIRS CLASS TEST - 16 a) 1 only Carbon credits are a part of attempts to mitigate b) 2 only the growth in concentrations of GHGs. c) Both 1 and 2 Carbon credits or carbon offsets can be acquired d) Neither 1 nor 2 through afforestation, renewable Answer: A energy, CO2 sequestration, methane Explanation: capture, buying from an exchange (carbon The Nobel Prize in Economic Sciences for 2019 credits trading) etc. was jointly awarded to Abhijit Banerjee, Esther Under Carbon Credits Trading mechanism Duflo, and Michael Kremer, for their countries that emit more carbon than the quota experimental approach to alleviating global allotted to them buy carbon credits from those poverty. Hence, statement 1 is correct. that emit less. Prof. Amartya Sen was the first Indian recipient Carbon credit prices are traded on an exchange. of the Nobel Prize for Economics for the year So, their prices are never fixed. 1998. He also became the first Asian to have been honoured with the award. Hence, statement 2 is 5. Consider the following statements not correct. regarding “Carbon Emission”. 1. United States is the highest emitter of 4. Regarding “carbon credits” which one of the carbon di oxide in the world. the following statements is not correct? 2. Paris agreement in 2015 set the limit of a) The carbon credit system was ratified in global warming to 2 degree Celsius conjunction with the Kyoto Protocol above pre-industrial level. b) Carbon credits are awarded to countries Which of the following statements is/are correct? or groups that have reduced greenhouse a) 1 only gases below their emission quota b) 2 only c) The goal of the carbon credit system is to c) Both 1 and 2 limit the increase of carbon emission d) Neither 1 and 2 quota Answer: B d) Carbon credits are traded at a price fixed Explanation: The top four emitters in 2017, from time to time by the United Nations which covered 58 per cent of global emissions, Environment Programme. were China (27 per cent), the US (15 per cent), Answer: D the European Union (10 per cent) and India (7 Explanation: Carbon credit – Kyoto Protocol per cent), according to the projection by the A carbon credit (often called a carbon offset) is Global Carbon Project. The rest of the world a tradable certificate or permit. contributed 41 per cent last year. 2015 global One carbon credit is equal to one tonne of carbon climate change agreement in Paris set the limit of dioxide.

www.iasgatewayy.com

UPSC OCTOBER- 2019 DAILY CURRENT AFFAIRS CLASS TEST - 16 global warming to 2 degrees Celsius above pre- Answer: A industrial levels. 8. Which of following becomes first UN 6. Consider the following with respect to organisation to hold and make transactions Enforcement Directorate: in cryptocurrencies ether and bitcoin? 1. It is part of the Department of Economic a) UNCTAD Affairs, Ministry of Finance. b) UNICEF 2. Its prime objective is the enforcement of c) UNESCO the Foreign Exchange Management Act d) WHO

1999 (FEMA) and the Prevention of Answer: B Money Laundering Act 2002 (PMLA). Which of the statement(s) given above is/are 9. Consider the following statements with correct? respect to DFDT insecticide. a) 1 only 1. DFDT kills mosquitoes including species b) 2 only that carry malaria, yellow fever, Dengue c) Both 1 and 2 and Zika. d) Neither 1 nor 2 2. It acts two to four times faster than Answer: B DDT.

Which of the statement(s) given above is/are 7. Consider the following with respect to correct? Ayushman Bharat – Pradhan Mantri Jan a) 1 only Arogya Yojana: b) 2 only 1. It is an umbrella of two major health c) Both 1 and 2 initiatives, namely Health and wellness d) Neither 1 nor 2 Centres and National Health Protection Answer: C Scheme.

2. It is a Centrally Sponsored Scheme.

3. It is being implemented by Central

government directly.

Which of the statement(s) given above is/are correct?

a) 1 and 2 only

b) 2 and 3 only

c) 1 and 3

d) All the above

www.iasgatewayy.com

UPSC OCTOBER- 2019 DAILY CURRENT AFFAIRS CLASS TEST - 16 10. Consider the following with respect to Bluetongue (BT) virus. 1. It is a mosquito-transmitted viral disease of domestic and wild ruminants. 2. It is wide spread among both cattle and human in India. Which of the statement(s) given above is/are correct? a) 1 only b) 2 only c) Both 1 and 2 d) Neither 1 nor 2

Answer: D

www.iasgatewayy.com

UPSC OCTOBER- 2019 DAILY CURRENT AFFAIRS CLASS TEST - 17 1. Consider the following statements with 2. “Corruption perception Index” was released respect to the objectives of “Make in India” by scheme. a) World Economic Forum 1. To increase the manufacturing sector’s b) Transparency International share in the economy from 16 to 25 per c) World Bank cent of the GDP by 2022. d) Organisation of Economic Cooperation 2. To create 100 million additional jobs by and Development 2022 in manufacturing sector. Answer: b) 3. Ensuring sustainability of growth, Explanation: Since its inception in 1995, the Corruption Perceptions Index, Transparency particularly with regard to environment. International’s flagship research product, has Which of the above statements is/are correct? become the leading global indicator of public a) 1 only sector corruption. b) 1 and 2 only The index offers an annual snapshot of the c) 1 and 3 only relative degree of corruption by ranking d) All of the above countries and territories from all over the globe. Answer: d) The 2018 CPI draws on 13 surveys and expert Explanation: Prime Minister launched the assessments to measure public sector corruption Make in India campaign on September 25, 2014. in 180 countries and territories, giving each a Achievable Targets: score from zero (highly corrupt) to 100 (very Target of an increase in manufacturing sector clean). growth to 12-14% per annum over the medium India ranks 78 out of 180 countries in term. Transparency International’s Corruption An increase in the share of manufacturing in the Perception Index. country’s Gross Domestic Product from 16% to

25% by 2022. 3. Which of the following country is not To create 100 million additional jobs by 2022 in manufacturing sector. bordered by Syria? Creation of appropriate skill sets among rural a) Turkey migrants and the urban poor for inclusive b) Iraq growth. c) Iran An increase in domestic value addition and d) Jordan technological depth in manufacturing. Answer: c) Enhancing the global competitiveness of the Indian manufacturing sector.

Ensuring sustainability of growth, particularly with regard to environment. www.iasgatewayy.com

UPSC OCTOBER- 2019 DAILY CURRENT AFFAIRS CLASS TEST - 17 4. Consider the following with respect to c) World Trade Organization Transports Internationaux Routiers (TIR) d) A and C 1. 1.The conventions were adopted under Answer: b) the auspices of the United Nations 7. Consider the following with respect to Economic Commission for Europe Ionosphere. (UNECE). 1. It contains electrically charged particles 2. India has ratified the convention. known as ions. So, it is known as Which of the statement(s) given above ionosphere. is/are correct? 2. It is located between 20km to 80km. a) 1 only 3. Here the temperature starts decreasing b) 2 only with increase in height. c) Both 1 and 2 Which of the statement(s) given above is/are d) Neither 1 nor 2 incorrect? Answer: c) a) All the above 5. Consider the following statements with b) 1 and 3 respect to the Index of Industrial c) 1 and 2 Production (IIP). d) 2 and 3 1. It is released in a quarterly basis by Answer: d) Central Statistical Organisation.

2. It divides industrial economy is divided 8. Which among the following was not an into three sectors, with highest economic activity performed between 2000 weightage to manufacturing. to 2010? Which of the statement(s) given above is/are a) Setting up Board for Reconstruction of correct? Public Sector Enterprises a) 1 only b) Establishment of Competition b) 2 only Commission of India c) Both 1 and 2 c) Establishment of National Disaster d) Neither 1 nor 2 Management Authority Answer: b) d) Setting up of Disinvestment Commission Answer: d) 6. Asia-Pacific Trade and Investment Report Explanation: Board for Reconstruction of is published by? Public Sector Enterprises (BRPSE) was a) U.S. Chamber of Commerce established in December 2004 as an advisory b) UNCTAD and UN Economic and Social body to advise the Government on the strategies, Commission for Asia and the Pacific measures and schemes related to strengthening,

www.iasgatewayy.com

UPSC OCTOBER- 2019 DAILY CURRENT AFFAIRS CLASS TEST - 17 modernizing, reviving and restructuring of public c) Both 1 and 2 sector enterprises. Composition. d) Neither 1 nor 2 Competition Commission of India is a statutory Answer: c) body of the Government of India responsible for • Radio Frequency Identification (RFID) enforcing The Competition Act, 2002 throughout technology uses electromagnetic fields to India and to prevent activities that have an automatically identify and track tags appreciable adverse effect on competition in containing electronically stored India. information. NDMA is an agency of the Ministry of Home • It uses radio waves to communicate Affairs whose primary purpose is to coordinate between two objects: a reader and a tag. response to natural or man-made disasters and • RFID communication is the same as two- way radio communication in the sense for capacity-building in disaster resiliency and that information is transmitted or crisis response. NDMA was established through received via a radio wave at a specific the Disaster Management Act 2005. frequency. Disinvestment Commission was set up in 1996.

9. With Which country is India conducting the 10th edition of the joint Military exercise ‘Vajra prahar 2019’? a) USA b) China c) Australia d) Indonesia

Answer: a)

10. Consider the following statements: 1. RFID technology uses electromagnetic fields to automatically identify and track tags containing electronically stored information. 2. RFID communication is the same as two-way radio communication. Which of the given statement/s is/are correct? a) 1 only b) 2 only

www.iasgatewayy.com

UPSC OCTOBER- 2019 DAILY CURRENT AFFAIRS CLASS TEST - 18 1. Consider the following statements a) 1 only regarding GOAL (Going Online as Leaders) b) 2 only programme: c) Both 1 and 2 1. It is a programme aimed at inspiring, d) Neither 1 nor 2 guiding and encouraging tribal girls to Answer: b) become leaders. Explanation: The Global Hunger Index report 2. It is a programme of Facebook which is is an annual publication that is jointly prepared by the Concern Worldwide and Welthungerhilfe. jointly implemented in India with the Hence, statement 1 is not correct. help of Ministry of Tribal Affairs. India’s performance has downgraded since the Choose the correct statement using the code last decade. India’s rank has slipped from 95th given below: position (in 2010) to 102nd (in 2019). Hence, a) 1 only statement 2 is correct. b) 2 only

c) Both 1 and 2 3. Consider the following statements d) Neither 1 nor 2 regarding International Monetary Fund Answer: c) Explanation: GOAL (Going Online as Leaders) (IMF): is a Facebook programme aimed at inspiring, 1. The International Monetary Fund (IMF) guiding and encouraging tribal girls to become is an inter-governmental organisation leaders. So, statement 1 is correct. established to stabilize the exchange The Ministry of Tribal Affairs and Facebook rate in the international trade. together will digitally mentor 5000 young 2. It has been created during the Bretton women in India’s tribal dominated districts. So, Woods Conference of 1944. statement 2 is also correct. 3. It publishes the “World Economic Outlook” report. 2. With reference to the ‘Global Hunger Index Choose the correct statement using the code Report’, consider the following statements: given below: 1. The report is annually released by the a) 1 and 3 only International Food Policy Research b) 1,2 and 3 only Institute (IFPRI). c) 1,3 and 4 only 2. India’s performance has been d) 2,3 and 4 only continuously declining since the last few Answer: b) years. Explanation: All the above statements are true Which of the statements given above is/are regarding IMF. So, option B is the answer. correct?

www.iasgatewayy.com

UPSC OCTOBER- 2019 DAILY CURRENT AFFAIRS CLASS TEST - 18 7. Consider the following statements with 4. Paektu Mountain is also known as Baekdu respect to Anti-Dumping Duty: Mountain is situated at the border between 1. It is a protectionist tariff that a domestic which of the following countries? government imposes on Foreign a) China and Mongolia Imports. b) Russia and China 2. It is imposed by Directorate General of c) North Korea and China Trade Remedies, under Ministry of d) China and Pakistan Answer: c) Commerce and Industries.

Which of the statement(s) given above is/are 5. Consider the following statements with correct? respect to Food Safety Mitra scheme: a) 1 only 1. It aims to support small and medium b) 2 only scale food businesses to comply with the c) Both 1 and 2 food safety laws. d) Neither 1 nor 2 2. mA FSM is an individual professional Answer: a) certified by FSSAI. 3. They have the power to inspect and even 8. Consider the following statements with seal the firms which not in compliance respect to Services Exports Promotion with food safety laws. Council (SEPC): Which of the statement(s) given above is/are 1. It is an apex trade body which facilitates incorrect? service exporters of India. a) 1 and 2 only 2. It is an advisory body and it acts as an b) 2 only interface between the Services Industry c) 3 only and the Government. d) All the above 3. It has recently brought out the India Answer: c) Intellectual Property, IP Guide in MIPCOM 2019. 6. KHON Ramlila is a masked form of Ramlila Which of the statement(s) given above is/are art, famous in which of the following correct? country? a) 1 and 2 only a) Thailand b) 2 and 3 only b) Vietnam c) 1 and 3 c) Nepal d) All the above d) Sri Lanka Answer: d) Answer: a)

www.iasgatewayy.com

UPSC OCTOBER- 2019 DAILY CURRENT AFFAIRS CLASS TEST - 18 9. Shaheen VIII is joint military exercise between which of the following countries? a) India and USA b) USA and China c) USA and Pakistan d) Pakistan and China Answer: d)

10. Consider the following statements with respect to World Intellectual Property Organization: 1. It is the global forum for intellectual property policy, services, information and cooperation. 2. It is an UN Specialized Agency. 3. World Intellectual Property Indicators (WIPI) report is one of its publications. Which of the statement(s) given above is/are correct? a) 1 and 2 only b) 2 and 3 only c) 1 and 3 d) All the above Answer: d)

www.iasgatewayy.com

UPSC OCTOBER- 2019 DAILY CURRENT AFFAIRS CLASS TEST - 19 1. Consider the following statements Which of the statement(s) given above is/are regarding Financial Action Task Force correct? (FATF): a) 1 only 1. It was an initiative of G7 countries to b) 1 and 2 only combat growing problem of money c) 3 only laundering. d) 1 and 3 only 2. India is not a part of FATF. Answer: c) Explanation: Angel Tax is a 30% tax that is 3. Currently Pakistan and Iran are in the levied on the funding received by startups from FATF Blacklist category. an ‘Angel Investor’. Which of the statement(s) given above is/are This tax was recently scrapped as it was correct? considered as a bane for the Indian Startups and a) 1 only young entrepreneurs. b) 2 and 3 only

c) 1 and 3 only 3. Kuki community recently seen in news does d) 1,2 and 3 only not belong to which of the following states? Answer: a) a) Arunachal Pradesh Explanation: FATF comprises over 39 b) Mizoram members’ countries including India. So, c) Manipur statement 2 is wrong. d) Meghalaya Currently North Korea and Iran are in FATF Answer: a) Blacklist category. So, statement 3 is wrong. Explanation: The Kukis are one of several hill

tribes within India, Bangladesh, and Burma. 2. Consider the following statements In India Kukis are found in the states of regarding the ‘Angel Tax’: Manipur, Assam, Nagaland, Meghalaya, Tripura 1. Angel Tax is a 20% tax that is levied on and in the state of Mizoram. the funding received by startups from an Angel Investor. 4. Consider the following statements with 2. Currently this tax has been reduced to respect to Dark matter and Dark energy: 15% as it would encourage the startup 1. Dark matter constitutes around 70% of companies and the entrepreneurs. the mass of the universe and dark 3. This tax is levied when startups receive energy constitutes 25% of the mass of funding at a valuation higher than its the universe. fair market value. 2. As they contain mass, they interact with the light.

www.iasgatewayy.com

UPSC OCTOBER- 2019 DAILY CURRENT AFFAIRS CLASS TEST - 19 Which of the statement(s) given above is/are 7. Consider the following statements with correct? respect to India Innovation Index (III) a) 1 only 2019: b) 2 only 1. Gujarat is the most innovative state. c) Both 1 and 2 2. It is released by NITI Aayog. d) Neither 1 nor 2 Which of the statement(s) given above is/are Answer: d) correct? a) 1 only 5. Consider the following statements with b) 2 only respect to Tuberculosis: c) Both 1 and 2 1. UN aims to eradicate TB by 2030, while d) Neither 1 nor 2 India aims to eradicate it by 2025. Answer: b) 2. Rifampicin is the first-line of drug used to treat TB. 8. Which of the following cities is located 3. India accounted for 27 per cent of the closest to the Indian landmass? world total. a) Jaffna Which of the statement(s) given above is/are b) Colombu correct? c) Hambantota a) 1 and 2 only d) Mannar b) 2 only Answer: d)

c) 3 only d) All the above 9. The Asia Pacific trade investment report Answer: d) 2019 is published by a) United Nations Economic and Social 6. DANX-19, a military exercise which is Commission for Asia and the Pacific recently seen in the news is related to? (ESCAP) a) An exercise between India and Denmark b) Trans Pacific Partnership (TPP) b) Naval exercise between India and c) Association of Southeast Asian Australia Nations (ASEAN) c) An exercise related to the defence of d) South Asian Association for Regional Andaman and Nicobar Islands Cooperation (SAARC) d) Trilateral exercise between India and Answer: a) Russia Explanation: The Asia-Pacific Trade and Answer: c) Investment Report 2019 was published by the

United

www.iasgatewayy.com

UPSC OCTOBER- 2019 DAILY CURRENT AFFAIRS CLASS TEST - 19 Nations Economic and Social Commission for Asia and the Pacific (ESCAP) and the United Nations Conference on Trade and Development (UNCTAD).

10. Sirumugai is a town well known for silk sarees, it belongs to which of the following state? a) Tamil Nadu b) Uttar Pradesh c) Karnataka d) Andhra Pradesh

Answer: a)

www.iasgatewayy.com

UPSC OCTOBER- 2019 DAILY CURRENT AFFAIRS CLASS TEST - 21 1. Consider the following statements: 2. Which of the following is a most likely 1. National Payments Corporation of India consequence of implementing the ‘Unified (NPCI) helps in promoting the financial Payments Interface (UPI)’? inclusion in the country. a) Mobile wallets will not be necessary for 2. NPCI has launched RuPay, a card online payments. payment scheme. b) Digital currency will totally replace the Which of the statements given above is/are physical currency in about two decades. correct? c) FDI inflows will drastically increase. a) 1 only d) Direct transfer of subsidies to poor people b) 2 only will become very effective c) Both 1 and 2 Answer: a) d) Neither 1 nor 2 Explanation: Unified Payments Interface (UPI) Answer: c) is a system that powers multiple bank accounts Explanation: National Payments Corporation into a single mobile application (of any of India (NPCI), an umbrella organisation for participating bank), merging several banking operating retail payments and settlement features, seamless fund routing & merchant systems in India. It has been incorporated as a payments into one hood. It also caters to the “Not for Profit” Company under the provisions of “Peer to Peer” collect request which can be Companies Act 1956. scheduled and paid as per requirement and convenience. Its objective is to provide infrastructure to the entire Banking system in India for physical as 3. Consider the following statements well as electronic payment and settlement regarding UN Human Rights Council systems. The Company is focused on bringing (UNHRC): innovations in the retail payment systems 1. It requires absolute majority to get through the use of technology for achieving elected to UNHRC. greater efficiency in operations and widening the 2. No member country can hold two reach of payment systems. consecutive terms in the council. 3. Currently India is not a member of RuPay is an Indigenously developed Payment UNHRC. System by NPCI – designed to meet the Which of the statements given above is/are expectation and needs of the Indian consumer, correct: banks and merchant eco-system. a) 1 only b) 1 and 2 only c) 2 and 3 only d) 1,2 and 3 only Answer: a)

www.iasgatewayy.com

UPSC OCTOBER- 2019 DAILY CURRENT AFFAIRS CLASS TEST - 21 Explanation: To become a member, a country 5. Consider the following statements: must receive the votes of at least 96 of the 191 1. The Global Tuberculosis (TB) Report is states of the UN General Assembly (an absolute published by World Health majority). Elections takes place through secret Organisation. ballot. So, statement 1 is correct. 2. According to the report, TB incidence per 1,00,000 population in India has The members are elected for a period of three increased significantly from 2017. years, with a maximum of two consecutive terms. 3. Globally, India accounts for the second They cannot enjoy third consecutive term. so, highest number of TB cases after China. statement 2 is incorrect. Which of the statements given above is/are

correct: India has been elected to the United Nations a) 1 only Human Rights Council (UNHRC) for a period of b) 1 and 2 only three years beginning January 1, 2019 as a part of c) 1 and 3 only Asia- Pacific category. So, statement 3 is d) 2 and 3 only incorrect. Answer: a)

Explanation: The World Health Organization 4. Consider the following statements (WHO) has recently released its 2019 edition of regarding Asbestos: the Global Tuberculosis (TB) Report. 1. It has been widely used in the The report provides a comprehensive and up-to- manufacturing and the cosmetic date assessment of the TB epidemic and progress industry. in the response at global, regional and country 2. Asbestos is the major cause for a levels for India. WHO has been publishing global progressive cancer called mesothelioma. TB report every year since 1997. Hence, Which of the statements given above is/are statement 1 is correct. correct: According to the report, India has performed a) 1 only better in countering the TB. b) 2 only TB incidence per 1,00,000 population has c) Both 1 and 2 declined from 204 (2017) to 199 (2018). Hence, d) Neither 1 nor 2 statement 2 is not correct. Geographically, most Answer: c) TB cases in 2018 were in the WHO regions of Explanation: Both the statements are correct. South-East Asia (44%), Africa (24%) and the So, option 3 is correct. Western Pacific (18%). Eight countries that

accounted for two thirds of the global total

www.iasgatewayy.com

UPSC OCTOBER- 2019 DAILY CURRENT AFFAIRS CLASS TEST - 21 include India (27%), China (9%) and Indonesia 8. Consider the following statements (8%). Hence, statement 3 is not correct. regarding ‘Anthrax’: 1. Anthrax is a zoonotic disease that 6. Plethora 2, sometimes seen in news is mainly affects the livestock animals. referred to 2. It is a viral disease that affects human a) A crater found in the red planet mars when they come in contact with the b) A protein that induces regeneration infected animals. c) A satellite launched by SpaceX 3. It is a non-curable disease and can only d) An active in the ring of fire. be prevented by antibiotics. Answer: b) Which of the statement(s) given above is/are Explanation: Plethora 2 is a protein which correct? helps to regenerate the primary and lateral roots a) 1 only of the plants. b) 1 and 2 only

This discovery ensures the quick repair of the c) 1 and 3 only plants which is damaged either by high wind or d) 1, 2 and 3 rain. Answer: a)

However, excess dosage of this protein beyond Explanation: Anthrax is a serious infectious the threshold level reduces the regeneration zoonotic disease caused by a bacterium known as potential. Bacillus anthracis.  Bacillus anthracis is a gram-positive, rod- 7. Bhashan Char Island is located in which of shaped bacterium that mainly affects the the following seas? livestocks. a) Arabian Sea  Although it is rare, people can get sick with b) Philippines Sea anthrax if they come in contact with c) Bay of Bengal infected animals or contaminated animal d) Indian Ocean products. Answer: c)  It can occur in four forms: skin, lungs, Explanation: Bhashan Char or Thengar Char is intestinal, and injection. an island under the jurisdiction of Bangladesh  Two months of antibiotics such as which emerged only 2 decades before in the Bay ciprofloxacin, levofloxacin, and doxycycline of Bengal. after exposure can prevent infection.  Anthrax vaccination is recommended for

people who are at high risk of infection.

www.iasgatewayy.com

UPSC OCTOBER- 2019 DAILY CURRENT AFFAIRS CLASS TEST - 21 9. The purpose of Jan Aushadhi Sugam, a mobile app recently launched by union government is a) To buy generic medicines from all medical shops b) To avail discounts from Jan Aushadi Kendras c) To access directions to Jan Aushadi Kendras d) To gather information about various diseases Answer: c) Explanation:  Jan Aushadhi Sugam” mobile application would have user-friendly options like- to locate nearby Jan Aushadhi Kendra, direction guidance for location of the Jan Aushadhi Kendra through Google Map, search Jan Aushadhi generic medicines, analyse product comparison of Generic and Branded medicine in form of MRP & overall Savings, etc.  The mobile application is available on both Android & iOS platforms.

10. National Security Guard (NSG) is functioning under which of the following? a) Ministry of Home Affairs b) Ministry of Defense c) Prime Minister Office d) President of India

Answer: a)

www.iasgatewayy.com

UPSC OCTOBER- 2019 DAILY CURRENT AFFAIRS CLASS TEST - 22 1. Consider the following statements 2. In the context of National Tiger regarding the Defence Acquisition Council Conservation Authority (NTCA) consider (DAC): the following statements: 1. The Chairman of DAC is the Minister of 1. It is a statutory body under the Defence and the Vice Chairman is the chairmanship of the Minister of Secretary of Defence. Environment, Forests and Climate 2. It was constituted to ensure a fair Change. defence procurement planning process. 2. It approves the reserve specific tiger 3. It also advices the Prime Minister conservation plan prepared by the regarding the national security policy. Central Government. Which of the statements given above is/are Which of the statements given above is/are Incorrect? correct? a) 1 only a) 1 only b) 1 and 2 only b) 2 only c) 1 and 3 only c) Both 1 and 2 d) 1, 2 and 3 d) Neither 1 nor 2 Answer: c) Answer: a) Explanation: The Chairman of DAC is the Explanation: The reserve specific tiger Minister of Defence and the other members conservation plan is prepared by the State include Minister of State for Defence, Chief of Government. So, statement 2 is not correct. Army Staff, Chief of Naval Staff, Chief of Air 3. Thylacine, a marsupial which was said to be Staff, Defence Secretary, Secretary of Defence extinct was found again in which of the Research & Development, Secretary of Defence following places? Production, Chief of Integrated Staff Committees a) Indonesia b) Tasmania HQ IDS, Director General (Acquisition), Dy. c) Brazil d) Madagascar Chief of Integrated Defence as Staff Member Answer: b) Secretary.

There is no Vice Chairman. So, statement 1 is not 4. Place of Effective Management (POEM) is correct. the term recently seen in the news is related National security policy advisory is done by to? National Security Advisor. So statement 3 is also a) Shell companies not correct. b) Non-Performing Assets

c) Ground water reserve

d) Mining Answer: a)

www.iasgatewayy.com

UPSC OCTOBER- 2019 DAILY CURRENT AFFAIRS CLASS TEST - 22 5. Consider the following statements with 8. Which of the following is Not one among respect to Inter- Parliamentary Union the seven social sins of Mahatma Gandhi? (IPU): a) Success without integrity 1. IPU is the global organization of b) Pleasure without Conscience national parliaments. c) Religion without sacrifice 2. It is under the United Nations. d) Knowledge without character 3. India is member of this organization. Answer: a) Which of the statement(s) given above is/are Explanation: incorrect? Mohandas Karamachand Gandhi, one of the a) 1 and 2 only most influential figures in modern social and b) 2 and 3 only political activism, considered these traits to be c) 2 only the most spiritually perilous to humanity. d) 1 and 3 only • Wealth without Work Answer: c) • Pleasure without Conscience • Science without Humanity 6. Okayama Declaration is related to, • Knowledge without Character a) Primary Education • Politics without Principle b) Climate change c) Cyber security • Commerce without Morality d) Health care • Worship without Sacrifice Answer: d) 9. Which one of the following is a very 7. Consider the following statements with significant aspect of the Champaran respect to PACEsetter Fund: Satyagraha? 1. It was constituted by India and the USA. a) Active all-India participation of lawyers, 2. It aims to promote artificial intelligence students and women in the National in India. Movement Which of the statement(s) given above is/are b) Active involvement of Dalit and Tribal correct? communities of India in the National a) 1 only Movement b) 2 only c) Joining of peasant unrest to India’s c) Both 1 and 2 National Movement d) Neither 1 nor 2 d) Drastic decrease in the cultivation of Answer: a) plantation crops and commercial crops Answer: c)

www.iasgatewayy.com

UPSC OCTOBER- 2019 DAILY CURRENT AFFAIRS CLASS TEST - 22 Explanation: The Champaran Satyagraha was (Marxist), the Revolutionary Socialist Party, and the combination of an elements of extra- the All India Forward Bloc – nominated Sahgal constitutional struggle as well as the employment as a candidate in the presidential elections. She of moral force against an adversary, an exemplar was the sole opponent of A.P.J. Abdul Kalam, of the rule of law; and the use of compromise as a who emerged victorious gambit.

It marked as the first India’s Civil Disobedience movement launched by Mahatma Gandhi to protest against the injustice meted out to tenant farmers in Champaran district of Bihar.

He led organised protests and strike against the landlords, who with the guidance of the British government, signed an agreement granting more compensation and control over farming for the poor farmers of the region, and cancellation of revenue hikes and collection until the famine ended. Source: Yojana

10. Identify the Freedom Fighter using the below 1. She lead the Women’s organisation of Azad Hind Government 2. Contested unsucesfully in one of India’s Presidential Elections a) Usha Mehta b) Laxmi Sehgal c) Janaky Athi Nahappan d) Pritilata Waddedar Answer: b) Explanation: She was a revolutionary of the Indian independence movement, an officer of the Indian National Army, and the Minister of Women's Affairs in the Azad Hind government. In 2002, four leftist parties – the Communist Party of India, the Communist Party of India

www.iasgatewayy.com

UPSC OCTOBER- 2019 DAILY CURRENT AFFAIRS CLASS TEST - 23 1. Consider the following statements a) National Human Rights Commission regarding TechSagar: b) Central Bureau of Investigation 1. It is a consolidated and comprehensive c) National Crime Record Bureau repository of India’s cyber tech d) National Institute of Criminology and capabilities. Forensic Science 2. This has been launched with the Answer: c) partnership of Data Security Council of Explanation: India (DSCI). National Crime Record Bureau (NCRB) brings Choose the correct statements using the code out its annual comprehensive statistics of given below: crime across the country in the ‘Crime in India’ a) 1 only report. b) 2 only The report is being published since 1953 and c) Both 1 and 2 serves as a crucial tool in understanding the d) Neither 1 nor 2 law and order situation across the country. Answer: c) Hence, option C is the correct answer. Explanation: It is a consolidated and comprehensive 3. Arrange the following glaciers from West to repository of India’s cyber tech capabilities East: which provides actionable insights about 1. Siachen glacier capabilities of the Indian Industry, academia 2. Gangotri glacier and research across 25 technology areas. So, 3. Zemu glacier statement 1 is correct. 4. Baltoro glacier

Which of the following is the correct sequence? National Cyber Security Coordinator’s office in a) 1-2-3-4 partnership with Data Security Council of India b) 2-3-1-4 (DSCI) has launched national repository of c) 4-3-2-1 India’s cyber tech capabilities named d) 4-1-2-3 TechSagar. Answer: d)

So statement 2 is also correct. Explanation: The Siachen Glacier (around 77° E) is located 2. ‘Crime in India’ report is an annual in the eastern Karakoram range in the publication of which of the following Himalayas. organization: Gangotri Glacier (around 79° E) is located in Uttarakhand, India in a region bordering Tibet.

www.iasgatewayy.com

UPSC OCTOBER- 2019 DAILY CURRENT AFFAIRS CLASS TEST - 23 Zemu Glacier (around 88° E) is located at the 5. Which of the following are the terms of base of in the Himalayan reference (TOR) given to 15th Finance region of Sikkim, India. Commission Baltoro glacier (around 76° E) is located in the 1. The efforts taken to promote digital Gilgit-Baltistan region of Pakistan. Hence, economy option D is the correct answer. 2. Eliminating losses of power sector 3. To end open defecation 4. With reference to the Financial Stability 4. Promoting ease of doing business and Development Council (FSDC), consider Which of the statement(s) given above the following statement. is/are correct? 1. It was set up to enhance inter-regulatory a) 1 and 4 only coordination b) 1,3 and 4 only 2. It is a statutory body c) 1 only 3. It was chaired by Governor of Reserve d) All of the above Bank of India Answer: d) Which of the statement(s) given above is/are Explanation: correct? The 15th Finance Commission was notified on a) 1 only November 27, 2017 and was mandated to b) 1 and 3 only suggest a formula for devolution of funds to c) 1 and 2 only states for five years beginning April 1, 2020.

d) All the above The Commission may consider proposing Answer: a) measurable performance-based incentives for Explanation: States, at the appropriate level of government, The Financial Stability and Development in following areas. Council (FSDC) were constituted in December, Promoting digital economy and removing 2010. The FSDC was set up to strengthen and layers between the government and the institutionalize the mechanism for maintaining beneficiaries. financial stability, enhancing inter-regulatory Progress made in sanitation, solid waste coordination and promoting financial sector management and bringing in behavioral development. change to end open defecation. An apex-level FSDC is not a statutory body. Progress made in promoting ease of doing The Council is chaired by the Union Finance business by effecting related policy and Minister regulatory changes and promoting labour

intensive growth.

www.iasgatewayy.com

UPSC OCTOBER- 2019 DAILY CURRENT AFFAIRS CLASS TEST - 23 6. Regarding, Taanaji Malusare maratha 7. Consider the following statements with military leader consider following respect to Hunar Haat statements. 1. It is a mission to provide a platform to 1. Taanaji is popularly remembered for the master artisans and craftsmen to display Battle of Singhagad that took place in as well sell their products. the year 1670. 2. It is organised by Ministry of Culture. 2. Taanaji was having rivalry with Which of the statement(s) given above is/are Chatrapati Shivaji. correct? 3. The fort of Kondhana was renamed a) 1 only as Singhagad (lion’s fort) to honour b) 2 only Tanaji. c) Both 1 and 2 Which of the statements given above is/are d) Neither 1 nor 2 correct? Answer: b) a) 1 and 2 only Explanation: Hunar haat comes under the b) 1 and 3 only Ministry of Minority affairs. c) 3 only Hunar Haat components: d) 1, 2 and 3  To promote marketing and sale of products Answer: b) of individual Minority craftsmen/ artisans. Explanation: Statement 2 is wrong.  To provide platform for the artisans and Who was Tanaaji Malusare? culinary expert belonging to minorities to He was a Maratha military leader and a close showcase and sale their exquisitely crafted aide of Chhatrapati Shivaji. products and serve traditional cuisine. Hailing from the Malusare clan, Taanaji is  To support participation of minority popularly remembered for the Battle of craftsmen/ artisans in exhibition, trade fair, Singhagad that took place in the year 1670. buyer seller meets etc., To support In the battle, Taanaji fought against Udaybhan organisations (Government organizations, Rathore, a formidable Rajput warrior, who was Registered societies/ trusts/ companies, put in charge of Fort Kandhana (later named PSUs) for organising exhibitions at Singhagad) by Jai Singh. state/District level. To establish linkages Why this question with Dilli Haat, Handicraft Emporiums A Bollywood movie is being made based etc., Financial norms: 100% funding will be on Subedar Taanaji Malusare. done by the Ministry. Implementations of Hunar haat will be done through National Minorities development & finance corporation.

www.iasgatewayy.com

UPSC OCTOBER- 2019 DAILY CURRENT AFFAIRS CLASS TEST - 23 shares, car, any vehicle or mode of transport 8. Which of the following releases Global that costs more than Rs 10 lakh. Wealth Report?

a) Credit Suisse Group b) Welthungerhilfe 10. Consider the following statements with c) World Economic Forum respect to Polio in India: d) World Bank 1. India’s attainment of polio-free status in Answer: a) 2014. Explanation: The Credit Suisse Group, a 2. Pulse Polio Initiative was started with Switzerland-based multinational an objective of achieving a hundred per investment bank, has released the 10th cent coverage under Inactivated Polio edition of its annual Global Wealth Report. Vaccine. Which of the statement(s) given above is/are 9. Consider the following statements with correct? respect to definition of StartUps a) 1 only 1. An entity shall be considered a start-up b) 2 only up to 10 years from its date of c) Both 1 and 2 d) Neither 1 nor 2 incorporation/registration. Answer: a) 2. An entity would be considered as a Explanation: India’s attainment of polio-free startup up to a turnover of Rs 100 crore. status in 2014 was coined by the World Health 3. A start-up cannot offer loans or Organization as “one of the most significant advances, other than those where achievements in public health,” and marked lending money is part of its business. not just India but the entire South East Asia Which of the statement(s) given above is/are Region being declared polio-free. correct? The last case of polio in the country was in a) 1 and 2 only b) 2 and 3only 2011.25 years of Pulse Polio Programme to be c) 1 and 3 only celebrated in October 2019. It aimed to d) All the above immunize children through improved social Answer: d) mobilization, plan mop-up operations in areas Explanation: Apart from the third point, a where poliovirus has almost disappeared and start-up cannot invest in a building or land maintain a high level of morale among the unless it is for its business or used by it for public. purposes of renting or held by it as stock-in- trade. A start-up cannot make any capital contribution to any other entity or invest in www.iasgatewayy.com

UPSC OCTOBER- 2019 DAILY CURRENT AFFAIRS CLASS TEST - 24 1. Consider the following about Marawah nickname "Sejal Kapoor" on Facebook used spy Island applications called "Whisper" and "Gravity Rat ". 1. It’s a Paleolithic Site off Dubai Coast 2. Recently, the oldest known human 3. Global Snow Leopard and Ecosystem skeleton was discovered here Program (GSLEP), sometimes seen in news Which of the above statement(s) is/are correct? comprises of which of the following a) 1 only countries? b) 2 only 1. Kazakhstan c) Both 1 and 2 2. Tajikistan d) Neither 1 nor 2 3. Turkmenistan Answer: d) 4. Kyrgyzstan Explanation: The oldest known natural pearl in 5. Uzbekistan the world has been discovered by Abu Dhabi Select the correct answer using the codes given archaeologists working at a Neolithic site on below: Marawah Island, just off the coast of Abu Dhabi. a) 1, 2, 3 and 4 only b) 1, 2, 3 and 5 only Dubbed the ‘Abu Dhabi Pearl’, it was found in c) 1, 2, 4 and 5 only layers that have been radiocarbon dated to d) All the above 5,800-5,600 BCE, during the Neolithic period. Answer: c) Explanation: GSLEP comprises of Russia, 2. Recently seen in news, Whisper and Gravity China, Mongolia, Kazakhstan, Kyrgyzstan, RAT refers to Uzbekistan, Tajikistan, India, Nepal, Bhutan, a) Plant Proteins Afghanistan and Pakistan. b) Malware Among the 5 Central Asian countries c) Exoplanets Turkmenistan is not present in the GSLEP. d) Gene Editing Techniques Answer: b) 4. Travel and Tourism Competitiveness Index Explanation: Detectives in UP Police and was released by which of the following? military intelligence (MI) have found a Pakistani a) UNESCO spy who uses robots to hack into computer b) World Economic Forum systems. Till date, he has hacked more than 98 c) UN World Tourism Organisation officials from various forces and defence d) World Travel and Tourism Council organizations, including the Indian Army, the Air Answer: b) Force, the Navy, paramilitary forces and state

police personnel in Rajasthan, MP, Punjab and UP from 2015 to 2018. The spy, who got the

www.iasgatewayy.com

UPSC OCTOBER- 2019 DAILY CURRENT AFFAIRS CLASS TEST - 24 5. Consider the following statements advertisement campaign 'Feed Our Future' to regarding ‘Snow Leopard’: raise awareness and take steps against hunger 1. It is the state animal of Uttarakhand. and malnutrition in India. 2. Hemis National Park in Uttarakhand is ‘Feed Our Future’ focuses on the urgent attention the natural habitat of the snow leopard. for the critical issue of hunger and malnutrition 3. Bishkek Declaration was adopted to in the country. protect the snow leopard and its So, statement 1 is correct and statement 2 is incorrect. environment.

Which of the statements given above are correct? 7. ‘Mission Indradhanush’ covers which of a) 1 and 2 only the following diseases: b) 2 and 3 only 1. Polio c) 1 and 3 only 2. Japanese Encephalitis d) 1, 2 and 3 3. Tetanus Answer: b) 4. Cancer Explanation: Snow Leopard is the state animal Choose the correct answer using the code given of Himachal Pradesh. below: Uttarakhand’s state animal is Alpine musk deer. a) 1 and 3 only

b) 2 and 4 only 6. Consider the following statements c) 1, 2 and 3 only regarding 'Feed Our Future' campaign: d) 1, 2, 3 and 4 1. This campaign is launched by United Nations World Food Programme (WFP). Answer: c) 2. The campaign is to promote the prey Explanation: It provides vaccination against base for the critically endangered 12 Vaccine-Preventable Diseases (VPD) i.e. species to ensure their longevity. diphtheria, Whooping cough, tetanus, polio, tuberculosis, hepatitis B, meningitis and Choose the correct answer using the code given pneumonia, Hemophilus influenza type B below: infections, Japanese encephalitis (JE), rotavirus a) 1 only vaccine, pneumococcal conjugate vaccine (PCV) b) 2 only and measles-rubella (MR). c) Both 1 and 2

d) Neither 1 nor 2 8. Aflatoxin-M1, recently in news is produced Answer: a) by: Explanation: The United Nations World Food a) Bacteria b) Fungi Programme (WFP) has joined hands with c) Protozoa d) Virus Bollywood in India and launched a cinema Answer: b)

www.iasgatewayy.com

UPSC OCTOBER- 2019 DAILY CURRENT AFFAIRS CLASS TEST - 24 Explanation: Recently the Food Safety and Answer: b) Standards Authority of India (FSSAI) has tested Explanation: Union Minister of State for different types of milk (raw and processed of Culture & Tourism (IC), Shri Prahlad Singh Patel various types) samples under the National Milk launched the E-Portal of CCRT ‘Digital Bharat Safety and Quality Survey 2018. It has revealed Digital Sanskriti’ and CCRT YouTube Channel that the problem of Aflatoxin-M1 was found in both processed milk than raw milk. Aflatoxins are toxins produced by certain fungi which are generally found in agricultural crops like maize, peanuts, cotton seed and others. They are carcinogenic in nature.

9. Colonel Chewang Rinchen Setu, recently in News, is in: a) Gulf of Kutchh b) Palk Strait c) Andaman and Nicobar Island d) Ladakh

Answer: d) Explanation: Defence Minister Rajnath Singh inaugurated the Col. Chewang Rinchen Setu which has been constructed at an altitude of 14,650 feet in the forward area of Ladakh region. This bridge has been built by the Border Roads Organisation (BRO).

10. ‘Digital Bharat Digital Sanskriti' is an initiative of a) MEITY b) Ministry of Culture c) Ministry of Human Resource Development d) Ministry of Inormation and Communication

www.iasgatewayy.com

UPSC OCTOBER- 2019 DAILY CURRENT AFFAIRS CLASS TEST - 25 1. Recently World Health Organization b) Bureau of Police Research and announced the eradication of which of the Development following? c) State Police departments a) Small Pox d) Central Vigilance Commission b) Wild poliovirus type 2 Answer: b) c) Wild poliovirus type 3 Explanation: Union Minister G Kishan Reddy d) Yaws releases Bureau of Police Research and Answer: c) Development’s (BPRD) flagship publication Data Explanation: In an historic announcement on on Police Organizations World Polio Day, an independent commission of 4. Consider the following with respect to Snow experts concluded that wild poliovirus type 3 Leopard: (WPV3) has been eradicated worldwide. 1. They inhabit alpine and subalpine zones

at elevations from 3,000 to 4,500 m. 2. Consider the following statements with 2. In 2017, status of Snow leopard in the respect to definition of Kartarpur Sahib Red List of the IUCN was changed from Corridor. ‘vulnerable’ to 'endangered'. 1. It stands on the bank of the River Ravi. 3. Government of India launched Project 2. It is the final resting place of Guru Leopard in 1992 to in the 5 Himalayan Nanak. states. 3. As per the agreement, Indian pilgrims of Which of the statement(s) given above is/are only Sikh faith can use the corridor. correct? Which of the statement(s) given above is/are a) All the above correct? b) 2 and 3only a) 1 and 2 only c) 1 and 3 only b) 2 and 3only d) 1 and 2 only c) 1 and 3 only Answer: c) d) 2 only

Answer: a) 5. Consider the following statements: Explanation: Indian pilgrims of all faith can 1. Convention for the Conservation of use the corridor. Antarctic Marine Living Resources is an

international commission with 26 3. Which of the following organizations recently released the Data on Police Members and India being one of them. Organization? 2. It was in response to increasing a) National Crime Records Bureau commercial interest in Antarctic krill resources.

www.iasgatewayy.com

UPSC OCTOBER- 2019 DAILY CURRENT AFFAIRS CLASS TEST - 25 3. Krill is a keystone component of the Versailles is a specialized agency of the UN. Arctic ecosystem. Hence, statement 2 is correct. Which of the statement(s) given above is/are 7. ‘6X6X6 strategy’ is related to which of the incorrect? following disease? a) 1 only a) Tuberculosis b) 2 and 3only b) HIV/AIDS c) 3 only c) Anaemia d) All the above d) Malaria Answer: c) Answer: c) Explanation: Krill is a keystone component of Explanation: The 6X6X6 strategy (six target the Antartic ecosystem. beneficiary groups, six interventions, and six

institutional mechanisms) is related to anaemia 6. Consider the following statements testing and treatment. Hence, option C is the regarding United nations: correct answer. 1. To become a member of the UN, the

States need to be admitted by the 8. With reference to the ‘Livestock Census’ in General Assembly on the India, consider the following statements: recommendation of the Security 1. It is conducted every 5 years to cover all Council. domesticated animals and their 2. ILO is a specialized agency of the UN. headcounts. Choose the correct statement using the code 2. The 20th census has been conducted by given below: the Ministry of Agriculture and Farmers a) 1 only Welfare. b) 2 only Which of the statements given above is/are c) Both 1 and 2 correct? d) Neither 1 nor 2 a) 1 only Answer: c) b) 2 only Explanation: The UN Charter declares that the c) Both 1 and 2 membership of the UN is open to all the peace- d) Neither 1 nor 2 loving nations who accept its obligations. The Answer: a) States are admitted by the General Assembly on Explanation: The Livestock Census has been the recommendation of the Security Council. conducted in the country periodically since 1919- Hence, statement 1 is correct. 20. Since then it has been conducted once every 5 The International Labour Organization (ILO) years and covers all domesticated animals and was also created in 1919 under the Treaty of their headcounts. Hence, statement 1 is correct.

www.iasgatewayy.com

UPSC OCTOBER- 2019 DAILY CURRENT AFFAIRS CLASS TEST - 25 The census is conducted by the Ministry of 10. Dr Syama Prasad Mookerjee Tunnel, Fisheries, Animal Husbandry, and Dairying, in recently seen in news, connects: participation with State Governments and UT a) Udhampur to Ramban in Jammu. Administrations. Hence, statement 2 is not b) Ukshi and Bhoke station. correct. c) Manali and Keylong. d) Khopoli to Lonavla. 9. Consider the following statements Answer: a) regarding the Universal Service Obligation Explanation: Union Minister for Road Fund (USOF): Transport & Highways announced renaming of 1. USOF was formed by the NITI Aayog to Chenani Nashri Tunnel on NH 44 in Jammu & help fund projects to boost connectivity Kashmir as Dr Syama Prasad Mookerjee Tunnel in urban as well as rural areas. in an event in New Delhi today. 2. It was envisaged in National Telecom This 9 km tunnel is the longest such state of art Policy, 1999 and was given statutory tunnel in the country, connecting Udhampur to status through Indian Telegraph Ramban in Jammu.

(Amendment) Act, 2003.

3. The resources are raised through a ‘Universal Access Levy (UAL)’, which would be a percentage of the revenue earned by the telecom operators. Which of the statement(s) given above is/are incorrect? a) 1 Only b) 1 and 2 only c) 1 and 3 only d) 1, 2 and 3 Answer: a) Explanation: Universal Service Obligation Fund (USOF) was formed by the Central Government to help fund projects to boost connectivity in rural areas only.

www.iasgatewayy.com

UPSC OCTOBER- 2019 DAILY CURRENT AFFAIRS CLASS TEST - 30

1. Kartarpur Corridor connects which of the 3. The term Bio-terrorism recently in news following pilgrimages of Sikhs? refers to a) Anandpur Sahib and Dera Baba Nanak a) Terrorism using naturally occurring Virus Sahib and Bacteria b) Dera Baba Nanak Sahib and Gurudwara b) Terrorism using modified forms of Virus Darbar Sahib and Bacteria c) Goindwal Sahib and Anandpur Sahib c) Both a and b d) Gurudwara Darbar Sahib and Goindwal d) None of the above Sahib Answer: c) Answer: b) Explanation: Explanation:  Bioterrorism is terrorism involving the  Kartarpur Corridor connects Dera Baba intentional release or dissemination of Nanak Sahib in the state of Punjab, India biological agents. and Gurudwara Darbar Sahib at Kartarpur  These agents are bacteria, viruses, fungi, or in Punjab province of Pakistan. toxins, and may be in a naturally occurring or a human-modified form, in much the 2. Consider the following statements same way in biological warfare. regarding the Competition Commission of  Recently Union defence minister described India (CCI): bio-terrorism as a “contagious plague” and 1. It is a statutory body established under asked the armed forces to be at the the Competition Act, 2002. forefront of combating the menace. 2. The Commission comprises of six members including the Chairperson. 4. Which reference to ‘World Banks’s Ease of Which of the statement(s) given above is/are doing business’, recently in the news, which correct? of the following statements is/are correct? a) 1 only 1. India went up 14 rungs in the 2020 b) 2 only survey to score a 63, making it the one c) Both 1 and 2 of world’s top 10 most improved d) Neither 1 nor 2 countries for the third consecutive time. Answer: a) 2. Ease of doing business measures Explanation: regulation on employing workers and Including the Chairperson, the commission contracting with the government. comprises of 7 members. Select the correct answer using the code given below:

www.iasgatewayy.com

UPSC OCTOBER- 2019 DAILY CURRENT AFFAIRS CLASS TEST - 30 a) 1 only  Newly independent nations dreamed they b) 2 only could make their way in this new world c) Both 1 and 2 without hewing to either of the big powers, d) Neither 1 nor 2 the United States and the ,

Answer: a) eschewing the icy hostilities of the Cold Explanation: Statement 2 is wrong War and bask in the warmth of Third World Doing Business also measures regulation on (as it was then known) cooperation. employing workers and contracting with the Why this Question government, which are not included in the ease  Prime Minister Modi will skip the meet. of doing business score and ranking. This is the second time in a row that Why this Question PM Narendra Modi will skip the summit, World Bank’s ease of doing business ranking marking India’s transformation from a non- released. aligned country to one which is 10 top improvers are Saudi Arabia, Jordan, Togo, supposedly multi-aligned. Bahrain, Tajikistan, Pakistan, Kuwait, China, India, and Nigeria. 6. Consider following statements regarding Taanaji Malusare, Maratha military leader 5. Consider the following statements with 1. Taanaji is popularly remembered for the reference to the ‘Non-Aligned Movement' Battle of Singhagad that took place in 1. The Asian-African Conference of 1955 the year 1670. held in Bandung was the catalyst for the 2. He had rivalry with Chatrapati Shivaji. establishment of the Non-Aligned 3. The fort of Kondhana was renamed Movement. as Singhagad (Lion’s Fort) to honour 2. None Aligned movement is formed in Tanaji. backdrop of Vietnam War, as to de Which of the statement(s) given above is/are associate for major powers. correct? Which of the statement(s) given above is/are a) 1 and 2 only correct? b) 1 and 3 only a) 1 only c) 3 only b) 2 only d) 1, 2 and 3 c) Both 1 and 2 Answer: b) d) Neither 1 nor 2 Explanation: Statement 2 is wrong. Answer: a) Who was Tanaaji Malusare? Explanation:  He was a Maratha military leader and a Statement 2 is wrong close aide of Chhatrapati Shivaji. www.iasgatewayy.com

UPSC OCTOBER- 2019 DAILY CURRENT AFFAIRS CLASS TEST - 30 Inhabit alpine and subalpine zones at elevations  Hailing from the Malusare clan, Taanaji is from 3,000 to 4,500 m (9,800 to 14,800 ft). popularly remembered for the Battle of State animal of Himachal Pradesh and Singhagad that took place in the year 1670. the National Heritage Animal of Pakistan.  In the battle, Taanaji fought against Habitat extends through twelve Udaybhan Rathore, a formidable Rajput countries: Afghanistan, Bhutan, China, India, warrior, who was put in charge of Fort Kazakhstan, Kyrgyzstan, Mongolia, Nepal, Kandhana (later named Singhagad) by Jai Pakistan, Russia, Tajikistan, and Singh. Uzbekistan. China contains as much as 60% of all Why this Question snow leopard habitat areas.  A Bollywood movie is being made based Listed on Appendix I of the Convention on on Subedar Taanaji Malusare. International Trade of Endangered Species (CITES). 7. With reference to ‘Snow leopard’ found in Global Snow Leopard Forum, 2013:12 countries India, consider the following statements: encompassing the snow leopard’s range 1. Snow Leopard is State animal of (Afghanistan, Bhutan, China, India, Kazakhstan, Himachal Pradesh and the National Kyrgyz Republic, Mongolia, Nepal, Pakistan, Heritage Animal of Pakistan. Russia, Tajikistan, and Uzbekistan). 2. Global Snow Leopard Forum, 2013 Bishkek Declaration: To protect the species and declares 12 countries encompassing the it’s environment. Global Snow Leopard and Eco-system Protection snow leopard’s range Afghanistan, Program: It is a joint initiative of range country Bhutan, China, India, Kazakhstan, governments, international agencies, civil Kyrgyz Republic, Mongolia, Nepal, society, and the private sector. Goal — secure the Pakistan, Russia, Tajikistan, and long-term survival of the snow leopard in its Uzbekistan. natural ecosystem. 3. Bishkek Declaration protects Snow Why this Question: Leopard species and its environment. Recently, First National Protocol to Enumerate Which of the statement(s) given above is/are Snow Leopard Population in India Launched. correct?

a) 1 and 2 only b) 3 only 8. Consider the following statements: c) 2 and 3 only d) 1, 2 and 3 1. Depletion of ozone is due to many Answer: d) factors, the most dominant of which is Explanation: the release of chlorine from HFCs Listed as Vulnerable on the IUCN Red List of (Hydrofluorocarbons) which destroys Threatened Species. the ozone.

www.iasgatewayy.com

UPSC OCTOBER- 2019 DAILY CURRENT AFFAIRS CLASS TEST - 30 2. The ozone layer exists mainly in the 2. During El Nino there is a warming of the stratosphere, a layer of the atmosphere sea surface temperature in the eastern that reaches from 10 to 50 kilometers and central equatorial Pacific Ocean. (about 6 to 30 miles) above the Earth’s Which of the statements given above is/are surface. correct? Which of the statement(s) given above is/are a) 1 only correct? b) 2 only a) 1 only c) Both 1 and 2 b) 2 only d) Neither 1 nor 2

c) Both 1 and 2 Answer: c) d) Neither 1 nor 2 Explanation: Answer: b) Both statements is right. Explanation: What is El Nino?  Statement 1 is wrong  El Nino is a climatic cycle characterised by  It is not Hydrofluorocarbon that damages high air pressure in the Western Pacific and Ozone layer but Chlorofluorocarbon that low air pressure in the eastern. damages Ozone layer  During this event, there is a warming of the  Statement 2 is right sea surface temperature in the eastern and Why this Question central equatorial Pacific Ocean.  During September and October, the annual  It is one phase of an alternating cycle ozone hole over the Antarctic reached its known as El Niño Southern Oscillation peak extent of 16. 4 million sq km on (ENSO). September 8, then shrank to less than 10 Why this Question million sq km. This has been the smallest  In a new study, researchers have found observed since 1982. NASA has described it that because of climate change, extreme El as great news for the Southern Hemisphere. Nino events are likely to become more frequent.

9. With reference to ‘El Nino phenomenon, 10. With reference to Model Agricultural consider the following statements: Produce and Livestock Marketing 1. El Nino is a climatic cycle characterised (Promotion & Facilitation) Act, 2017, by high air pressure in the Western consider the following statement. Pacific and low air pressure in the 1. It aimed to establish a regulated eastern. wholesale agri-market at a distance of every 80 km.

www.iasgatewayy.com

UPSC OCTOBER- 2019 DAILY CURRENT AFFAIRS CLASS TEST - 30

2. It has proposed to issue licenses to new private players and traders. Which of the statement(s) given above is/are correct? a) 1 only b) 2 only c) Both 1 and 2 d) Neither 1 nor 2

Answer: c) Explanation:  The new model law seeks to establish a regulated wholesale agri-market at a distance of every 80 km. To enable this, it has proposed to issue licenses to new private players and traders who wish to set up a wholesale market.  Unlike the current system, now only by paying unified single fees, farmer/trader will be able to transact in all such regulated agri-markets within the state.  It caps market fee (including developmental and other charges) at not more than 1 per cent for fruit and vegetables, and two per cent for food grain. It caps commission agents’ fee at not more than two per cent for non-perishables and four per cent for perishables.  Besides, it stipulates a single license for trading within the state and at the national level.

www.iasgatewayy.com

UPSC OCTOBER- 2019 DAILY CURRENT AFFAIRS CLASS TEST - 31

1. Consider the following pairs: of External Affairs in September 1964. The Islands Located at River programme is fully funded by the Government of 1. Bashan Char Island - Meghna India. Hence, statement 1 is not correct. Programme SCAAP (Special Commonwealth 2. Majuli Island - Brahmaputra African Assistance Programme) is a corollary 3. Omkareshwar Island - Narmada programme of the ITEC. Under it, 161 countries Which of the pairs given above is/are correctly from Asia, Africa, East Europe, Latin America, matched? the Caribbean as well as Pacific and Small Island a) 1 and 2 only countries are invited to share in the Indian b) 2 and 3 only developmental experience in various fields c) 1 and 3 only acquired over six decades of India's existence as a d) 1, 2 and 3 free nation. Hence, statement 2 is correct. Answer: d) Explanation: 3. Consider the following statements: Bashan Char Island is located at Meghna river. 1. SARAS Aajeevika Mela is an initiative of Majuli Island is located at Brahmaputra river. the Ministry of Skill Development & Omkareshwar Island is located at Narmada river. Entrepreneurship. Hence, option D is the correct answer. 2. The Mela is an integrated approach

towards women empowerment. 2. With reference to ‘Indian Technical and Which of the statements given above is/are Economic Cooperation (ITEC) Programme’, incorrect? consider the following statements: a) 1 only 1. It is a programme of the Ministry of b) 2 only Human Resource Development. c) Both 1 and 2 2. The SCAAP programme is an offshoot of d) Neither 1 nor 2 the ITEC. Answer: a) Which of the statements given above is/are Explanation: correct? SARAS Aajeevika Mela is an initiative by the a) 1 only Deendayal Antyodaya Yojana-National Rural b) 2 only Livelihoods Mission (DAY-NRLM) under the c) Both 1 and 2 Ministry of Rural Development. Hence, d) Neither 1 nor 2 statement 1 is not correct. Answer: b) The Mela aims to bring the rural women Self Explanation: Help Groups (SHGs) under one platform in order The Indian Technical and Economic Cooperation to showcase their skills and sell their products, (ITEC) programme was launched by the Ministry and thereby acts as an integrated approach www.iasgatewayy.com

UPSC OCTOBER- 2019 DAILY CURRENT AFFAIRS CLASS TEST - 31 towards women empowerment. Hence, 5. Which of the following is/are NOT statement 2 is correct. inflammatory bowel disease/s (IBD): 1. Crohn’s disease 4. Consider the following statements 2. Ulcerative colitis regarding Pradhan Mantri Fasal Bhima 3. Jaundice Yojana (PMFBY) Choose the correct option: 1. It provides insurance coverage and a) 1 only financial support to farmers b) 3 only 2. Post-harvest losses are included in the c) 1 and 3 only scheme. d) 1, 2 and 3 3. There is a uniform premium for kharif Answer: b) and Rabi crops. Explanation: Which of the above statements is/are correct? Inflammatory bowel disease (IBD) is an umbrella a) 1 and 2 only term used to describe disorders that involve b) 1 and 3 only chronic inflammation of the digestive tract. c) 2 and 3 only Types of IBD include: d) All of the above Ulcerative colitis: This condition causes long- Answer: a) lasting inflammation and sores (ulcers) in the Explanation: innermost lining of the large intestine (colon) The PMFBY will replace the existing two schemes and rectum. National Agricultural Insurance Scheme as well Crohn’s disease: This type of IBD is characterized as the Modified NAIS. It aims to provide by inflammation of the lining of the digestive insurance coverage and financial support to the tract, which often spreads deep into affected farmers in the event of failure of any of the tissues. notified crop as a result of natural calamities, The most common areas affected by Crohn’s pests & diseases. disease are the last part of the small intestine and There will be a uniform premium of only 2% to the colon. be paid by farmers for all Kharif crops and 1.5% The exact cause of Crohn’s disease remains for all Rabi crops. In case of annual commercial unknown. Previously, diet and stress were and horticultural crops, the premium to be paid suspected, but now doctors know that these by farmers will be only 5%. factors may aggravate but don’t cause Crohn’s In post-harvest losses, coverage will be available disease. A number of factors, such as heredity up to a maximum period of 14 days from and a malfunctioning immune system, likely play harvesting for those crops which are kept in “cut a role in its development. & spread” condition to dry in the field.

www.iasgatewayy.com

UPSC OCTOBER- 2019 DAILY CURRENT AFFAIRS CLASS TEST - 31 Both ulcerative colitis and Crohn’s disease 7. The term pegasus recently seen in news is usually involve severe diarrhea, abdominal pain, related to: fatigue and weight loss. a) A Roman Goddess excavated in Turkey b) Nuclear attack submarine developed by 6. Consider the following statements: Israel 1. Bhaona is a traditional form of c) A Spyware which infested WhatsApp entertainment created by Sankardeva. d) None of the above 2. The art form is widely practiced in the Answer: c) north eastern state of Assam. Explanation: 3. They are one act plays written in Facebook-owned platform WhatsApp revealed Brajawali and Sanskrit. that journalists and human rights activists in Which of the given statement/s is/are incorrect? India have been targets of surveillance by a) a. 1 and 2 only operators using Israeli spyware Pegasus. b) b. 3 only Pegasus sends people “exploit links.” Once a user c) c. 1 and 3 only clicks on it, the malware can penetrate a phone’s d) d. 1, 2 and 3 security features, and Pegasus is installed on the

Answer: b) device without the owner’s knowledge or Explanation: permission. Once in the system, Pegasus can Bhaona is a traditional form of entertainment, extract the user’s private data, including always with religious messages, prevalent in passwords, contact lists, calendar events, text Assam. messages, and even voice calls.

It is a simple drama which shows the victory of 8. Consider the following statements truth at the climax, generally using fictitious or regarding Lymphatic filariasis (LF): mythological kings, queens, demons, Gods, 1. It is a neglected tropical disease soldiers, etc. commonly known as elephantiasis The history of Assamese drama begins with the 2. It is a fatal disease which is likely to kill plays of Mahapurusha Srimanta Sankardeva 8 of the 10 affected people (1449-1568) written in the early sixteenth 3. It is caused mainly by Wuchereria century. Bancrofti and spread by Culex mosquito These plays are popularly known as Ankiya Nats Which of the statements given above is/are (one act play) and their staging is known as correct? bhaona. a) 1, 2 and 3 The bhaonas are written in the Assamese and b) 2 and 3 only Brajavali languages. c) 1 and 2 only

d) 1 and 3 only

www.iasgatewayy.com

UPSC OCTOBER- 2019 DAILY CURRENT AFFAIRS CLASS TEST - 31 Answer: d) 2. Any Indian between ages of 18 to 65 Explanation: years may voluntarily join the NPS. Lymphatic filariasis (LF) is a neglected tropical 3. Overseas Citizens of India (OCI) are not disease commonly known as elephantiasis. eligible to join NPS. LF disease is caused mainly by Wuchereria Which of the following statement(s) is/are Bancrofti and spread by Culex mosquito. This correct? mosquito grows in dirty accumulated water. a) 1 only Infection occurs when filarial parasites are b) 1 and 2 only transmitted to humans through mosquitoes. The c) 2 and 3 only infection is usually acquired in childhood, d) 1, 2 and 3 causing hidden damage to the lymphatic system. Answer: b) LF does not kill the affected people, but may Explanation: cause permanent disfigurement, reduced Recently PFRDA has notifies that even OCI are productivity and social stigma. Therefore 2nd eligible to join the NPS. statement is incorrect.

9. ‘Perseids’ sometimes seen in news is

referred to a) Meteor shower b) Crater in moon c) Radio galaxy d) Newly discovered satellite

Answer: a) Explanation: The Perseids are one of the brighter meteor showers of the year. They occur every year between July 17 and August 24 and tend to peak around August 9-13.

10. Consider the following statements regarding the National Pension Scheme (NPS): 1. National Pension System (NPS) is a government-sponsored pension scheme that is managed by PFRDA.

www.iasgatewayy.com